You are on page 1of 24

SEPTEMBER 2021 | VOLUME 18 | ISSUE 9

PEDIATRIC

Emergency Medicine Practice Evidence-Based Education • Practical Application

CLINICAL CHALLENGES
• What are the typical presentations of
common pediatric envenomations?

• For which types of envenomations is


antivenom available, and what are the
indications for its use?

• Which patients need observation,


and how long should the observation
period be?

Authors
Michael Levine, MD, FACEP, FACMT
Associate Professor of Emergency Medicine,
University of California, Los Angeles, Los Angeles, CA

Nathan Friedman, MD
Department of Emergency Medicine, University of
Terrestrial Envenomations
California, Los Angeles, Los Angeles, CA
in Pediatric Patients:
Peer Reviewers Identification and
Sing-Yi Feng, MD
Associate Professor, Division of Emergency
Medicine, Department of Pediatrics, Medical
Management in the
Toxicologist, North Texas Poison Center, University
of Texas Southwestern Medical Center, Dallas, TX
Emergency Department
Nicole Gerber, MD
Assistant Professor of Clinical Emergency Medicine n Abstract
and Clinical Pediatrics, Department of Emergency The majority of bites and stings from terrestrial animals are not
Medicine, Division of Pediatric Emergency
dangerous. However, due to their smaller size, children may be
Medicine, New York-Presbyterian/Weill Cornell
Medical Center, New York, NY more susceptible to the effects of venom, and they may experi-
ence more-severe envenomation effects than adults. This issue
reviews the basic epidemiology and underlying pathophysiol-
ogy of the bites and stings of spiders, bees and wasps, fire
Prior to beginning this activity, see the ants, scorpions, snakes, and lizards. Clinical presentations are
“CME Information” on page 21. reviewed, and evidence-based recommendations are provided
for management of the envenomated patient. While the patho-
physiology and much of the presentation and treatment are
similar for both children and adults, there can be subtle differ-
ences, which will be highlighted in this review.

For online access, scan with your


smartphone camera or QR code reader app:

This issue is eligible for 4 CME credits. See page 21. EBMEDICINE.NET
Case Presentations
An 8-year-old boy presents after a rattlesnake bite on his right ankle...
• The boy was hiking with his parents when he was bitten by a rattlesnake. He says the pain began almost
CASE 1

instantly.
• On examination in the ED, he has edema extending from the mid-foot to proximal to the knee. The calf
compartments are soft. Ecchymosis is noted as well as oozing from the 2 puncture wounds.
• You order laboratory studies and wonder if antivenom administration is warranted.

A 9-year-old girl presents after she felt a “pinch” on her arm while raking leaves...
• While en route to the hospital, the girl developed worsening pain at the site and localized sweating
over the affected extremity.
CASE 2

• In the examination room, the girl is crying, appears to be in great discomfort, and has abdominal
cramping. Your examination is notable for a tiny puncture mark surrounded by 2 to 3 cm of erythema
over the right arm but minimal edema.
• Based on these findings, you are concerned that this is a black widow bite. You recall that there is an
antivenom, but you are not sure whether it would be the best choice for this patient.

A 17-month-old boy presents after awakening crying in his crib shortly after midnight...
• The boy resides in Arizona and was in his usual state of health when he went to sleep. The parents note
CASE 3

that the boy is drooling, flailing all of his extremities, and has “funny” eye movements.
• On examination, no bite or sting marks are appreciated.
• Based on the findings and the geographical location, you are concerned for scorpion envenomation
and wonder how to confirm the diagnosis.

n Introduction stopping progression of swelling and reversing system-


In 2018, the American Association of Poison Control ic and other hematologic effects of the venom, but was
Centers received more than 13,000 calls for bites and associated with significant hypersensitivity reactions,
stings involving patients aged <20 years.1 Due to un- some of which were immediate (eg, anaphylactoid
derreporting, it is widely believed that these numbers reactions) and some of which were delayed (eg, serum
significantly underestimate the true prevalence. Ad- sickness).2 Consequently, a safer option was sought,
ditionally, because of the relative rarity of these events which led to production of antivenom that contains
and the potential for misidentification of the animal, Fab or F(ab’)2 fragments. Immunoglobulin G (IgG) is
the exact prevalence is not known. While many of these a large protein that is divided into an Fc (fragment,
incidents involve relatively minor symptoms, severe tox- crystallizable) portion and a Fab (fragment, antigen
icity and hospitalization are not uncommon, and, rarely, binding) portion.2 Enzymatic cleavage of IgG with pa-
some envenomations can be fatal. pain can generate 2 Fab fragments, whereas cleavage
Neither the underlying mechanisms of action of with pepsin yields a V-shaped F(ab’)2 fragment.3 Such
the toxin nor the treatment recommendations differ enzymatic cleavage allows isolation of the Fab portion,
substantially between pediatric and adult patients. which retains the antigen-specific binding component,
However, due to their smaller size, children may be while eliminating the most immunogenic parts of the
more susceptible to the effects of the venom, and immunoglobulin. Because Fab and F(ab’)2 are small
they may experience more-severe envenomations molecules, they have a distribution that mirrors the ex-
than adults, especially in the case of scorpion or hon- tracellular fluid and have a relatively rapid elimination.
eybee envenomation. Most antivenoms in the United States today are either
In an effort to minimize severe effects, antivenom Fab or F(ab’)2.
is often used after envenomation. In the United States, This issue of Pediatric Emergency Medicine
antivenom is available for black widow spiders, bark Practice focuses on the mechanism of action of the
scorpions, pit vipers (copperheads, cottonmouths, rat- various toxins of common envenomating terrestrial
tlesnakes), and coral snakes. Historically, most antive- creatures and provides recommendations for the clini-
noms were made from whole immunoglobulin. Wyeth cal evaluation and management of pediatric patients
North American coral snake antivenom was effective at who have been envenomated by spiders, various

SEPTEMBER 2021 • www.ebmedicine.net 2 ©2021 EB MEDICINE


stinging insects, scorpions, snakes, and lizards. Indica- vides questions to ask while taking the history.
tions for antivenom use and dosing recommendations In the absence of a clear history of a spider
are provided. bite, alternative etiologies should be considered, as
For a detailed review of marine toxins, see the numerous medical conditions can be misdiagnosed
April 2020 issue of Pediatric Emergency Medicine as spider bites. Most commonly, abscesses (particu-
Practice titled “Identification and Management of Ma- larly those from methicillin-resistant Staphylococcus
rine Envenomations in Pediatric Patients,” available aureus) are thought to be spider bites by patients.
at: www.ebmedicine.net/marine-envenomations.4 Conditions that can be misdiagnosed as loxoscelism
(systemic symptoms from envenomation by a Loxos-
celes reclusa [brown recluse] spider) include infec-
n Critical Appraisal of the Literature tions (bacterial, fungal, parasitic, and viral), necrotiz-
The literature search was comprised primarily of a ing vasculitis, lymphomatoid papulosis, pyoderma
PubMed search. Specific search terms used were gangrenosum, and pemphigus.5 Black widow bites on
envenomation, spider, Hymenoptera, bee, wasp, the abdomen have been misdiagnosed as an ab-
vespid, scorpion, rattlesnake, snake, and Heloderma. dominal pathology, such as appendicitis, because of
Abstracts from several national toxicology meetings, the intense pain surrounding the bite. Scorpion stings
including the Scientific Meetings of the American Col- might be perceived as an intoxication rather than an
lege of Medical Toxicology and the annual meeting of envenomation, due to the symptoms of altered men-
the North American Congress of Clinical Toxicology, tal status and roving eye movements. Determining
were reviewed for the most current literature. Most anaphylactic reactions versus direct venom reaction
of the literature focused on retrospective reviews and from massive envenomation (anaphylactoid reactions)
consensus guidelines. A few randomized clinical trials can be a challenge in patients who have been stung
were identified, which primarily focused on antive- by hymenoptera (bees, wasps, ants, etc). While ana-
noms. Many of the randomized trials are evidence- phylactic reactions are IgE-mediated, anaphylactoid
based, but many of the clinical practice recommenda- reactions are not. Fortunately, treatment for both are
tions are based on consensus. similar, and determination of the specific etiology can
be delayed until the patient is stabilized.

n Differential Diagnosis
The differential diagnosis of envenomations can be n Prehospital Care
quite broad, as patients often do not know what type of Prehospital care should focus on ensuring adequate
creature bit or stung them. Furthermore, differentiating airway, breathing, and circulation. Analgesics can
venomous bites and stings from nonvenomous bites be administered for pain; fentanyl is preferred, due
and stings can be difficult. The geographic location to minimal histaminergic effects.6 The immediate
where the envenomation occurred may offer some clues management of patients with massive Hymenoptera
as to what the envenomating creature was. Table 1 pro- envenomation is to remove the individual from the
swarm. For non-snake envenomations, the extremity
should be placed in a position of comfort. Pressure
Table 1. Components of the History in the
immobilization and tourniquets are contraindicated
Assessment of Terrestrial Bites and Stings for North American snake bites.7 For pit viper en-
• Species of biting or stinging creature (eg, bee, wasp, hornet, ant,
venomation, the extremity should be splinted and
scorpion, snake) or description of creature, if species unknown
• Location where bite or sting occurred, or activity being undertaken at
elevated; the splint should be applied loosely and
the time of the bite or sting posteriorly, to maintain the extremity in full exten-
• Time elapsed since bite or sting or onset of symptoms sion. Patients should be transported in a position of
• Progression of symptoms since bite or sting occurred comfort, ideally to an emergency department (ED)
• Site of bite or sting on the body
with capabilities to care for the patient. For snake
• Usual appearance of the affected area
• First aid measures already taken or treatments received
bites specifically, this should be to a facility that car-
• History of previous bites or stings (how many, location, etc), reaction ries adequate doses of the recommended antivenom.
to those stings (symptoms, length of reaction, treatment), and any In cases of a foreign body, there is no urgent need
additional symptoms for removal, and removal should not delay care or
• Allergy history including any medications used and any history of
transportation to an ED.
serious allergy, such as anaphylaxis, Stevens-Johnson syndrome,
serum sickness
• Medication history (particularly corticosteroids, other immune-
suppressing drugs, anticoagulants) n Spiders
• Other medical conditions, or chronic or recent/acute episodes of Despite spider bites being relatively common, hu-
illness
man toxicity is rare. Many spiders are too small to
• Recent surgical procedures or indwelling devices around area of the
bite or sting
be able to inject venom through human skin. Addi-
www.ebmedicine.net tionally, most spiders produce only a small amount

GROUP SUBSCRIPTIONS: groups@ebmedicine.net 3 © 2021 EB MEDICINE. ALL RIGHTS RESERVED.


of venom at a time, and the venom may not result neurotransmitter reuptake in the synapse, with effects
in significant physiologic activity in humans. There- on the sympathetic and parasympathetic nervous
fore, although many spiders are venomous, most systems.9,10 The toxin results in exocytosis of neu-
are not a threat to humans.8 rotransmitters involved in both calcium-mediated and
Laboratory studies are typically not indicated for non–calcium-mediated pore formation.
routine management of spider envenomations. There
are, however, a few exceptions. In cases of increased Clinical Presentation
neuromuscular activity due to Latrodectus mactans The initial Latrodectus bite is often painful, and a
(black widow spider) envenomation, creatine kinase “target” lesion (which consists of an area of central
levels and renal function should be monitored. In pallor surrounded by erythematous rings) may be
cases of L reclusa envenomation, if there is concern present. (See Figure 2.) The pain of the bite itself
for hemolysis, laboratory studies including bilirubin, may be localized to the site of the bite, but en-
complete blood cell count (CBC), lactate dehydroge- venomation often causes severe muscle pain and
nase, haptoglobin, creatinine, and free hemoglobin spasms of the back, abdomen, and chest muscles.
studies are recommended. Among patients with these lesions, approximately
25% develop latrodectism,6 a syndrome character-
Latrodectus: Widow Spiders ized by pain, sympathomimetic effects, diaphore-
Epidemiology sis, fasciculations, and back and/or limb pain. The
Spiders belonging to the genus Latrodectus, com- diaphoresis and muscle spasms can be diffuse or can
monly known as the widow spiders, are common be confined to the region of the envenomation.10-12
worldwide and can be found in Japan, New Zealand, Atypical manifestations include priapism, fever, car-
Australia, South America, and North America, as diomyopathy, and paresthesias.6,10,12,13 Pain and lo-
well as southern Europe/western Asia.9 In the United cal paresthesias generally develop rapidly, whereas
States, black widow spiders are indigenous to all systemic manifestations may be delayed for several
of the 48 contiguous states. There are >30 differ- hours. Symptoms generally peak during the first 48
ent known species of Latrodectus. The female black hours and decline within 2 to 3 days but can persist
widow spider is black, with a 12- to 16-mm thorax for up to a week or longer.
and a shiny red hourglass shape on the ventral side.
(See Figure 1.) The male black widow is smaller and
is not capable of causing human envenomations. Figure 2. Black Widow Spider Bite
Appearance
Pathophysiology
The primary toxin produced by the Latrodectus spe-
cies is alpha-latrotoxin, a neurotoxin that binds the
presynaptic neuron, resulting in depolarization of the
neuromuscular junction. In addition, the toxin prevents

Figure 1. Female Black Widow Spider


(Latrodectus mactans)

Republished with permission of McGraw Hill LLC, from Strange and Christos Gogos, Vasileios Sachpekidis, Acute myocardial injury caused
Schafermeyer's Pediatric Emergency Medicine, 5th edition, Milton by black widow spider (Latrodectus) bite, European Heart Journal -
Tenenbein, Charles G. Macias, Ghazala Q. Sharieff, et al, Copyright Case Reports, 2020, volume 4, issue 3, pages 1-2, by permission of
2019; permission conveyed through Copyright Clearance Center, Inc. Oxford University Press.

SEPTEMBER 2021 • www.ebmedicine.net 4 ©2021 EB MEDICINE


Treatment from the brown recluse has been “diagnosed” in
Treatment for Latrodectus envenomation is primar- virtually every state in the United States, including
ily supportive, consisting of benzodiazepines and many states in which the spider does not reside.8,17,18
opioids. In cases of severe toxicity, administration of Because toxicity from these bites primarily manifests
whole equine-based IgG antivenom can be consid- as necrotic dermal lesions, various soft-tissue infec-
ered. This antivenom, which is no longer produced, tions can be misdiagnosed as envenomation from this
can be obtained from the manufacturer on a case-by- spider.5 The bites most commonly occur on the trunk,
case basis for refractory cases. Its use remains quite thigh, or upper arm;12 bites on the hands, feet, neck,
controversial in the toxicology community, however, or face are rare.
due to a small number of fatal cases of anaphylaxis Fatalities from L laeta (Chilean recluse), found
following its administration.14 Use of this antivenom is primarily in South America, are well described. In the
generally not considered first-line therapy because of United States, fatal cases have been reported from
the relatively high rate of immediate (anaphylactoid) envenomation by L reclusa, although the exact preva-
or delayed (serum sickness) allergic reactions. In a lence has been debated and appears to be much less
recent review of Latrodectus envenomations in the common than its South American counterpart.12,17,19
United States, no deaths were ascribed to the enven-
omation itself, although deaths have occurred due to Pathophysiology
administration of the antivenom.11 Of note, F(ab’)2 There are several different components to Loxosceles
antibody-based antivenom is currently being devel- venom. The primary component, sphingomyelinase,
oped, with preliminary data that suggest efficacy in is responsible for the dermonecrotic lesions.6,17 In
reducing pain and spasms from envenomation, al- addition, the effect of sphingomyelinase D on the
though the antivenom has not yet obtained approval red blood cell membrane activates metalloproteases,
from the United States Food and Drug Administration ultimately leading to hemolysis.17
(FDA).15 Discussion with a medical toxicologist either
on staff or at a Poison Control Center (United States Clinical Presentation
National Poison Center telephone number: 1-800- After a bite from a Loxosceles spider, 1 of 4 events
222-1222) is recommended prior to administering can occur: (1) no effects are seen; (2) pain and ery-
Latrodectus antivenom. thema are present without systemic manifestations
In the past, it was thought that intravenous (IV) or sequelae; (3) dermonecrotic injury occurs with
calcium gluconate should be administered. However, eschar formation; or (4) systemic manifestations oc-
due to a lack of data demonstrating its efficacy, this cur.8,17 The initial bite is often painless, and patients
treatment is no longer recommended.16 may not immediately be aware of the bite.17 Over
the next several hours, erythema, pruritus, and a
Disposition burning pain may develop. The bite area itself can
Patients bitten by Latrodectus spiders may need to be become pale in appearance, and the surrounding
admitted if pain control is not achieved with oral medi- tissues may become erythematous and edematous.
cations. Additionally, if symptoms persist, admission More-severe envenomations can develop a central
may be indicated for titration of parenteral benzodi- blue-gray wound accompanied by a bull’s eye lesion
azepines and opioids. Otherwise, it is reasonable to
discharge the patient after an observation period of
several hours to ensure that there is no progression of Figure 3. Brown Recluse Spider
symptoms. (Loxosceles reclusa)
Loxosceles: Recluse Spiders
Epidemiology
L reclusa is commonly referred to as the brown
recluse or fiddleback spider because of the unique
shape of a violin on its cephalothorax. (See Figure
3.) These spiders are light or dark brown, with an
8- to 15-mm thorax. They live primarily in small,
irregular webs and are frequently found in boxes,
furniture, and basements, or under rocks or wood.
There are approximately 140 species of Loxosceles,
found worldwide.17 In the United States, they are
found primarily in the south and southeast, from
Texas to Georgia and the Gulf of Mexico to Missouri
and southern Illinois. They are most active at night Content provider: CDC/ Margaret Parsons. Available at: https://phil.cdc.
and generally bite when threatened.17 Envenomation gov/Details.aspx?pid=1125

GROUP SUBSCRIPTIONS: groups@ebmedicine.net 5 © 2021 EB MEDICINE. ALL RIGHTS RESERVED.


with an erythematous center, a white ring of indura- Treatment
tion, and an outer cyanotic ring,8,12,17 sometimes Most lesions from recluse spiders require no specific
referred to as a red, white, and blue appearance. intervention. In cases of wounds with large cutaneous
(See Figure 4.) Hemorrhagic vesicles can form over lesions, local wound care should be provided, and the
the next several days following the bite and prog- patient’s tetanus vaccination status should be updated,
ress to a central area of necrosis and ulceration. as indicated. Urgent excision of the wound is not
An eschar may form and ultimately dehisce over recommended, as it may result in prolonged recovery
several weeks.8,9,12,17 times and possibly worsened cosmetic outcomes. If an
Rarely, systemic manifestations (known as sys- eschar does develop, no surgical intervention should
temic loxoscelism or cutaneous-hemolytic loxosce- be performed until after the lesion has become well
lism) may occur. Systemic manifestations of Loxos- demarcated. In such cases, delayed surgical interven-
celes envenomation are more common in pediatric tion (including skin grafting) may be needed, but this
patients than adults.20 Early manifestations include typically should not be performed for several weeks to
low-grade fever and arthralgia,20 followed by gastro- months after the initial envenomation.
intestinal complaints, disseminated intravascular co- There are also inadequate data to recommend
agulation, and hemolysis. Rhabdomyolysis and renal routine prophylactic antibiotics, vasodilators, anti-
failure typically develop 48 to 72 hours after the en- histamines, corticosteroids, hyperbaric oxygen, or
venomation, although earlier cases have been docu- polymorphonuclear cell inhibitors (eg, dapsone or
mented.20 If systemic involvement is suspected, serial colchicine).6,12 While dapsone was once used, it is no
hemoglobin levels and plasma free haptoglobin longer recommended due to low-quality evidence of
levels should be obtained. Other laboratory markers its effectiveness in this setting. The use of systemic cor-
of hemolysis (including lactate dehydrogenase and ticosteroids should be considered in cases of systemic
indirect bilirubin levels) as well as serial creatinine loxoscelism, although data are somewhat conflicting.
and creatine kinase levels should be obtained. The
presence of hemoglobinuria should raise concern for Disposition
systemic manifestations.20 Patients with suspected cutaneous loxoscelism can be
Laboratory studies are not routinely indicated treated in the outpatient setting. If a patient has any
for all Loxosceles envenomations, however, and systemic symptoms, however, observation in a moni-
should be reserved for individuals with systemic tored setting for at least 24 hours is recommended.
complaints, such as weakness, fatigue, and pallor. Patients who lack systemic manifestations in the ED
The diagnosis of cutaneous loxoscelism is clinical. It following the initial bite can be discharged home with
is imperative that the patient is located in the cor- strict return precautions, including returning immedi-
rect geographic distribution of the spider and that ately should any systemic manifestations occur.
other etiologies, such as infectious etiologies, have
been excluded prior to making the diagnosis of L
reclusa envenomation. n Hymenoptera
The order Hymenoptera contains 3 medically impor-
tant families: Apidae (honeybees and bumblebees),
Vespidae (yellow jackets, wasps, and hornets), and
Figure 4. Brown Recluse Spider Bite
Formicidae (fire ants). (See Figure 5, page 7.)
Appearance
Bees and Wasps
Epidemiology
Hymenoptera envenomations are some of the most
common in the United States. Hymenoptera are
fairly passive animals, and they avoid humans unless
disturbed. Paper wasps and bumble bees live in hives
containing 100 to 200 insects, and they are not often
involved in mass envenomations. Social wasps (which
include hornets and yellow jackets) and honeybees
can be involved in massive envenomations, especially
if their colonies are threatened.
Yellow jackets are 10 to 15 mm in length, whereas
hornets are 15 to 40 mm in length. These insects re-
side in colonies of several hundred to a few thousand
Reproduced from BMJ Case Reports, Rare case of dermonecrosis
insects. The stingers of vespids (eg, yellow jackets,
caused by a recluse spider bite in Europe, Monique Cachia, Liam
Mercieca, Charles Mallia, et al, volume 2016, copyright 2016, with wasps, and hornets) and bumblebees are not barbed,
permission from BMJ Publishing Group Ltd. and the insect can pull its stinger out of flesh after

SEPTEMBER 2021 • www.ebmedicine.net 6 ©2021 EB MEDICINE


stinging without killing itself, allowing bumblebees occur as a result of an IgE-mediated anaphylactic
and vespids to sting multiple times. reaction, presenting with classic diffuse urticarial
Honeybees (Apis mellifera) generally reside in hives, bronchoconstriction, and hypotension. Deaths
colonies consisting of approximately 40,000 worker due to anaphylaxis are more common than anaphy-
bees and 1 queen bee. The stinger of a honeybee lactoid reactions. Because massive envenomations
is barbed, causing it to remain in mammalian flesh. result in anaphylactoid reactions, pediatric patients
Because the stinger of the honeybee is attached to its are somewhat more predisposed to life-threatening
abdomen, any attempt at removing the stinger results envenomations, as there is a relatively greater
in disarticulation of that portion of the abdomen and amount of venom in each sting on a per-kilogram
death of the honeybee. body-weight basis.
The Africanized honeybee (Apis mellifera scutel-
lata) was originally introduced in Brazil in an effort to Pathophysiology
make honey manufacturing more effective. In 1957, The venom of virtually all Apis species (eg, honeybee
the bees from 26 hives escaped and have gradu- and bumblebee) is similar, with small variations in
ally migrated north.21 Africanized insects are slightly composition. The primary component of honeybee
smaller, but, unlike their non-Africanized counterpart, venom is melittin, which causes toxicity by inserting
they frequently swarm in large groups. In addition, itself into the phospholipid bilayer of the cell mem-
these bees are capable of long flights without stop- brane, resulting in the destruction of red blood cells,
ping and will often attack following minimal provoca- platelets, and the vascular endothelium. In addition
tion. While the venom components are not signifi- to melittin, the phospholipase A2 enzyme is another
cantly different from their European counterparts, the prominent component of honeybee venom and is
large number of stings associated with the Africanized considered the primary allergen.13,21,22 Hyaluronidase,
honeybees’ swarming makes them potentially fatal, as another component, can increase tissue permeabil-
the cumulative venom dose is much greater.21 African- ity.13,21 Other components cause histamine release
ized honeybees are attracted to carbon dioxide and and inflammatory reactions.
selectively target the head and neck region. Other than anaphylactic reactions in patients with
Most cases of human toxicity from bee stings a prior immune reaction and allergy to honeybees,

Figure 5. Hymenoptera: Bees, Wasps, Hornets, and Fire Ants

A B C

D E F
A. Honeybee (Apis mellifera carnica). Source: https://commons.wikimedia.org/wiki/File:Apis_mellifera_carnica_worker_hive_entrance_2.jpg Author:
Makro Freak, Richard Bartz, Munich Makro Freak & Beemaster Hubert Seibring. Used under the Creative Commons Attribution-Share Alike 2.5
Generic license. https://creativecommons.org/licenses/by-sa/2.5/deed.en
B. Bumble bee. Source: https://commons.wikimedia.org/wiki/File:The_flight_of_the_bumble_bee_(8692773501).jpg Author: marsupium photography.
Used under the Creative Commons Attribution-Share Alike 2.0 Germany license. https://creativecommons.org/licenses/by-sa/2.0/deed.en
C. Wasp (Vespula vulgaris). Source: https://commons.wikimedia.org/wiki/File:Vespula_vulgaris5.jpg Author: Holger Gröschl. Used under the Creative
Commons Attribution-Share Alike 2.0 Generic license. https://creativecommons.org/licenses/by-sa/2.0/de/deed.en
D. Hornet (Vespa crabro). Source: https://commons.wikimedia.org/wiki/File:Vespa_crabro-lateral.jpeg Author: Niek Williems. Used under the Creative
Commons Attribution-Share Alike 2.5 Generic license. https://creativecommons.org/licenses/by-sa/2.5/deed.en
E. Paper Wasp (Polistes dominulus). Source: https://commons.wikimedia.org/wiki/File:Polistes_dominulus_fg01.JPG Author: Fritz Geller-Grimm. Used
under the Creative Commons Attribution-Share Alike 2.5 Generic license. https://creativecommons.org/licenses/by-sa/2.5/deed.en
F. Red Fire Ant (Solenopsis invicta). Adobe Stock Photo: 305823900.

GROUP SUBSCRIPTIONS: groups@ebmedicine.net 7 © 2021 EB MEDICINE. ALL RIGHTS RESERVED.


most envenomations result in local pain, erythema, antagonists [1 mg/kg IV or oral diphenhydramine] and
and edema. Massive envenomations, however, can H2 antagonists [0.5 mg/kg/dose famotidine every 12
result in significant systemic manifestations regard- hours, max 40 mg]), corticosteroids, and epinephrine.
less of prior immune reaction or exposure. Based on For pediatric patients who have between 2 and
animal models, it has been estimated that the median 50 stings (or <2 stings/kg), laboratory parameters
lethal number of stings is 18 stings per kilogram body should be obtained at presentation and at 6 hours af-
weight.23 However, deaths have been reported with ter presentation. These laboratory tests should assess
as few as 125 stings.24 Severe systemic toxicity can for renal injury, hepatic injury, cardiac injury, rhabdo-
occur with far fewer stings.24 Systemic manifestation myolysis, hemolysis, and coagulopathy.
after massive envenomations can include gastrointes-
tinal complaints (eg, vomiting and diarrhea). Third- Disposition
spacing of fluids and edema can occur, with resultant Patients with systemic manifestations should be admitted
hypotension and shock. Hemolysis, rhabdomyolysis, to an intensive care unit (ICU). Because of the potential
hepatic injury, renal failure, cardiac toxicity, and for delayed onset of many symptoms, a child who has
multisystem organ failure can ensue.21,24 Importantly, ≥50 honeybee stings (or ≥2 stings/kg for children weigh-
many of these manifestations may not be apparent ing <25 kg) should be admitted to a monitored setting
immediately, and may develop within 8 to 12 hours. for 24 hours.6 If renal injury, hepatic injury, cardiac injury,
rhabdomyolysis, hemolysis, or coagulopathy are pres-
Treatment ent, the patient should be admitted to an ICU, regardless
Research has demonstrated that nearly all of a bee’s of the number of stings. Patients who suffer systemic
venom is injected within 30 seconds of the sting.25 manifestations as a result of an anaphylactic reaction (but
Therefore, the initial management of victims of bee not necessarily from a massive envenomation) should be
envenomations should not focus on removal of the given a prescription for an epinephrine autoinjector and a
stingers. In the past, it was suggested that a credit referral to an allergist upon discharge.
card or other object should be used to scrape out
the stinger (rather than pulling it out with tweezers Fire Ants
or forceps) to avoid further release of venom. How- Epidemiology
ever, because it is now recognized that the stinger is The majority of fire ants that envenomate humans and
essentially deplete of venom by the time the patient affect crops are red fire ants (Solenopsis invicta), which
reaches the hospital, the method of removing the were accidentally introduced to Alabama from Brazil in
stinger is inconsequential.25 Although removal need the 1940s. These ants migrated out of Alabama, and
not be a priority step in management, stingers are a are now found throughout the southern United States,
foreign body and potential irritant, and they should from Virginia to California.26 Fire ant stings may be
be removed when the patient is stabilized. lethal due to either anaphylaxis or massive envenom-
The first step in management of Hymenoptera ation. As with other Hymenoptera, fatalities due to
envenomation is to ensure that the patient has a pat- anaphylaxis often occur shortly after the envenom-
ent airway. Facial edema may develop and, if there is ation, whereas death from massive envenomation may
concern for airway compromise, endotracheal intuba- occur more than 24 hours after the event.27
tion may be required. Aggressive resuscitation should Fire ants are typically 3 to 4 mm in length,
be provided with IV crystalloid fluids at a starting dose although the queen fire ant can be up to 1 cm in
of 20 to 40 mL/kg, up to a maximum of 2 L. Additional length. Mounds with multiple queens can contain
fluid therapy should be guided by clinical response. up to 500,000 worker fire ants. Unlike other Hyme-
Systemic corticosteroids (eg, 1-2 mg/kg of IV noptera that primarily attack when threatened, fire
methylprednisolone) should be administered as soon ants are aggressive without provocation. Attacks on
as possible. Patients with hypotension or evidence livestock and wildlife are common, and indoor attacks
of shock should receive epinephrine, which can be on debilitated humans (eg, nursing home patients)
administered either as an intramuscular (IM) injection are increasingly being reported.28,29 Prior to stinging,
(0.15 mg for patients weighing 15-30 kg; 0.3 mg for the fire ant bites its prey, locking its mandibles into
patients weighing ≥30 kg) or as a continuous IV infu- the soft tissue of the prey. The fire ant then stings the
sion (starting dose 0.1 mcg/kg, titrating up). Typically, victim and withdraws the stinger (the stinger is not
2 IM doses are attempted before IV epinephrine is barbed). While remaining attached to the victim with
initiated. Antihistamines and analgesics should be the strong mandible, the fire ant will rotate its body
administered based on the patient’s symptoms. It is and sting again multiple times.27
important to recognize that, regardless of whether a
patient is experiencing an anaphylactic reaction or an Pathophysiology
anaphylactoid reaction, the immediate treatment is The venom of fire ants is a water-insoluble alkaloid.26
essentially the same. Hymenoptera envenomations are The primary components of the venom include a
best treated with a combination of antihistamines (H1 hyaluronidase, a phospholipase, and the enzyme

SEPTEMBER 2021 • www.ebmedicine.net 8 ©2021 EB MEDICINE


N-acetyl-beta-glucosaminidase. The venom has administered as well. See the “Treatment” section for
hemolytic, cytotoxic, and neurotoxic components that Bees and Wasps on page 8 for dosing information.
inhibit sodium-potassium adenosine triphosphatase,
reduce cellular respiration, and uncouple oxidative Disposition
phosphorylation. In addition, the venom can result Patients with ongoing pain should be observed for
in impaired neutrophil and platelet function, which 6 hours to ensure that systemic symptoms do not
activates the coagulation system and results in a develop. Patients with severe pain or systemic mani-
hypercoagulable state.29 festations should be observed for 12 to 24 hours.
If no further symptoms develop, the patient can be
Clinical Presentation discharged home.
Following envenomation, a characteristic pattern of
toxicity can be observed. A local wheal and flair char-
acterized by a central edematous papule surrounded n Scorpions
by a pink-colored halo can occur within 20 minutes of Epidemiology
the exposure.28,30 This lesion is typically 25 to 50 mm While there are >1400 species of scorpions worldwide,
in diameter. Approximately 2 hours after the appear- only 30 are capable of producing clinically relevant
ance of the wheal, vesicles or pustules can develop. envenomation. The only scorpion in the United States
The fluid inside the vesicles is initially clear, but that is potentially lethal is the bark scorpion (Centru-
becomes cloudy within 8 hours; although cloudy, the roides sculpturatus). (See Figure 6.) This scorpion,
contents of the pustule are typically sterile.26 Necrosis which resides primarily in Arizona, is a tan-brown color
can develop within 24 hours. Some patients can de- and ranges from 1.3 to 7.6 cm in length.31 It is the only
velop large local reactions that are IgE-mediated and North American scorpion capable of walking up a ver-
mast cell-dependent. Occasionally, these lesions are tical wall. The scorpion body consists of 4 pairs of legs,
large enough to cause vascular compromise.30 Histo- a pair of grasping claws, mouthparts, a thorax, and a
logically, a classic triad can be observed, consisting of tail. The tail is a 6-segmented structure extending from
a subepidermal pustule, dermal neutrophilic infiltrate, the abdomen and terminating in a pointed telson that
and basophilic collagen degeneration.26 contains paired venom glands and the stinger.
While anaphylactic reactions can occur in pre- While scorpion stings are quite common in the
disposed individuals who are stung a single time, southwestern United States, most cases result in only
systemic manifestations of massive fire ant envenom- minor toxicity. Despite stings to adults being more
ation typically require a minimum of 50 to 100 stings. common, children are disproportionately severely en-
These manifestations are a direct response to the venomated and are more likely than adults to require
venom, and include rhabdomyolysis, disseminated in- intensive supportive care.32 In the 1930s, there were
travascular coagulation, and seizures. Diffuse urticarial 40 deaths reported due to scorpion envenomations,
rash, bronchoconstriction, and cardiovascular collapse with most of these cases occurring in infants and
can also occur.27 children.33 Due to improvements in access to care as
Following envenomation by multiple fire ants,
serum laboratory tests such as creatine kinase and a dis-
seminated intravascular coagulation profile (prothrom-
bin time, platelets, fibrinogen, D-dimer, and/or fibrin Figure 6. Bark Scorpion (Centruroides
split products) may be indicated to assess for rhabdo- sculpturatus)
myolysis or disseminated intravascular coagulation.
While exact recommendations are lacking, it is prudent
to check laboratory studies on patients with multiple
bites, especially pediatric patients, elderly patients,
and any patient with systemic involvement.

Treatment
Treatment for cutaneous symptoms is largely support-
ive. Antihistamines, topical corticosteroids, and cool
compresses are often used. Topical or subcutaneous
lidocaine may be used for patients with severe pain.
Patients with systemic toxicity should receive aggressive
supportive care in an ICU. Hypotension should be treat-
ed with IV fluid resuscitation (eg, 20 mL/kg crystalloid
fluids for pediatric patients), as well as with epinephrine Source: https://commons.wikimedia.org/wiki/File:Bbasgen-scorpion-front.
as either an IM injection or as a continuous IV infusion. jpg. Author: Musides. Used under the Attribution-ShareAlike 3.0 Unported
Systemic corticosteroids and antihistamines should be license. https://creativecommons.org/licenses/by-sa/3.0/deed.en

GROUP SUBSCRIPTIONS: groups@ebmedicine.net 9 © 2021 EB MEDICINE. ALL RIGHTS RESERVED.


well as the medical care itself, deaths due to scorpion Treatment
envenomations are currently extremely rare.1 Historically, antivenom therapy consisted of whole
IgG derived from goat serum. However, its antibody
Pathophysiology structure caused high immunogenicity with both im-
The venom of the bark scorpion is a complex mixture mediate (anaphylactoid) and delayed (serum sickness)
of various proteins, including neurotoxins that act on hypersensitivity reactions.36 Manufacturing of this
the sodium channels during their depolarized state, antivenom was discontinued in 2001. After remaining
resulting in inactivation of the channel. The end result supplies of the antivenom were exhausted, support-
is a prolonged action potential and increased and ive care was the only option available to patients
repetitive axonal firing that causes increased release outside of clinical trials. During the time when anti-
of acetylcholine and catecholamine.31 venom was not available, 20% of pediatric patients
presenting with high-grade scorpion envenomations
Clinical Presentation in Arizona required endotracheal intubation for re-
Following a scorpion sting, most patients exhibit spiratory failure.36 In August 2011, an equine-based
relatively minor symptoms, primarily local pain that Centruroides (scorpion) immune F(ab’)2 antivenom
typically presents immediately (if present), and the (Anascorp®) was approved by the FDA for the treat-
symptoms progress over the next hour. Because the ment of severe scorpion envenomations. The recom-
venom lacks dermonecrotic components, the sting mended initial starting dose of Anascorp® is 3 vials,
site is often not visible. In cases of diagnostic uncer- with additional vials given as needed. The number of
tainty, a tap test (in which tapping the envenomated vials is independent of the patient’s weight; thus, the
area elicits a painful reaction) can assist in establishing starting dose is the same for both children and adults.
the diagnosis. However, a retrospective study found that 1-vial serial
A grading system has been developed to classify dosing was as effective as 3-vial full dosing, with chil-
scorpion envenomations and aid in the determination of dren in both groups having resolution of symptoms
which patients should receive antivenom. (See Table 2.) within 4 hours of therapy initiation. No additional
A grade I envenomation results in pain and/or pares- complications or adverse clinical outcomes were ob-
thesias confined to the envenomation site. A grade II served in the 1-vial serial dosing group.37
envenomation has pain and/or paresthesias both at Because of the high cost of Anascorp®, some cli-
the sting site as well as in surrounding tissues. Grades nicians may prefer aggressive supportive care rather
III and IV envenomations are characterized by the pres- than antivenom administration.36 Supportive care
ence of either cranial nerve or skeletal muscle dysfunc- includes parenteral opioid analgesics for pain (eg,
tion (grade III), or both cranial nerve dysfunction and fentanyl 1-2 mcg/kg IV) in conjunction with benzodi-
skeletal muscle dysfunction (grade IV).32,34 azepines for neuromuscular hyperactivity (eg, mid-
Most patients with high-grade envenomations azolam 0.01-0.05 mg/kg IV). While most patients who
have tachycardia, hypertension, and diaphoresis. Vom- are envenomated develop minimal—if any—symp-
iting occurs in approximately one-third of patients with toms, the clinician treating patients with high-grade
severe toxicity, and it typically occurs relatively early in envenomations must be prepared to provide aggres-
the clinical course and resolves spontaneously.33 Cra- sive supportive care that may include endotracheal
nial nerve abnormalities include tongue fasciculations, intubation with mechanical ventilation.
opsoclonus-like roving eye movements, and bulbar
muscle dysfunction resulting in impaired swallowing. Disposition
Drooling is common, and aspiration is possible.6,34,35 Patients with North American scorpion stings can
Commonly encountered neuromuscular abnormalities be discharged if all of the following criteria are met:
include fasciculations, opisthotonos, ataxia, and flailing (1) symptoms are controlled with oral medication or
of the extremities.6,31,35 symptoms resolve after antivenom administration,
(2) vital signs have normalized, and (3) there are no
clinical findings of aspiration following an observation
Table 2. Scorpion Envenomation period of several hours. Patients who have persistent
Grading32 symptoms or those with severe symptoms who do not
receive antivenom should be admitted to the hospital
Grade Signs/Symptoms for further treatment and observation.
I Pain and/or paresthesias confined to the envenomation site
II Pain and/or paresthesias both at the sting site as well as in
surrounding tissues n Snakes
III Either cranial nerve dysfunction or skeletal muscle Epidemiology
dysfunction Worldwide, venomous snakes account for more cases
IV Both cranial nerve dysfunction and skeletal muscle of severe morbidity and mortality than any other
dysfunction animal. It is estimated that nearly 2.5 million individu-

SEPTEMBER 2021 • www.ebmedicine.net 10 ©2021 EB MEDICINE


als are bitten by venomous snakes annually, with paralysis would be observed. Most Mojave rattle-
125,000 individuals dying as a result of these enven- snakes contain venom B, and thus lack any significant
omations.38 There are approximately 3000 unique neurotoxicity. The Southern Pacific rattlesnake (C
species of snakes, of which 600 are venomous and oreganus helleri) can also produce neurologic toxicity,
200 are medically important.38 Venomous snakes that including ataxia and dysarthria.
are indigenous to the United States include Crotali- Various metalloproteinases are associated with
nae (pit vipers) and Elapidae (coral snakes). The North the hemorrhagic effects in Crotalinae venom. These
American pit vipers (named for the heat-sensing metalloproteinases destroy the capillary basement
pit on the head of the snake) include copperheads, membrane and capillary endothelial cells, ultimately
rattlesnakes, and cottonmouths. In the United States, resulting in edema and ecchymosis;42 platelets then
it is estimated that 9000 individuals will be bitten by occlude damaged capillaries. The venom can also
venomous snakes annually, resulting in 5 deaths.39 affect the coagulation cascade, resulting in coagu-
The North American Snakebite Registry (NASBR), a lopathy. Because the venom does not activate factor
national database of prospectively collected data on XIII, fibrin cross-linking does not occur, and the body’s
snakebites, reported 450 snakebites across 10 states endogenous fibrinolytic system degrades the ineffec-
between 2013 and 2015, almost all of which were pit tive clot, resulting in hypofibrinogenemia and fibrino-
viper envenomations.40 Rattlesnakes are responsible lysis.42 Thrombocytopenia can occur relatively early
for the majority of the envenomations and for the after envenomation, due to platelet aggregation,
greatest amount of morbidity and mortality. sequestration, and consumption.43
While snake venoms are quite heterogeneous Patients with rattlesnake envenomations can
among different species, they all serve 3 main pur- also develop late hematologic toxicities,44,45 such
poses: (1) acquisition of prey, (2) digestion of prey, as recurrent or delayed-onset thrombocytopenia
and (3) defense from real or perceived threats. Venom and/or coagulopathy. Recurrent hematologic
contains low-molecular-weight proteins and peptides toxicities are characterized by abnormal hematologic
that are directly toxic to certain cell types and enzymes laboratory findings during the index hospitalization
that break down specific tissues or cell membranes. that improved with antivenom, to again become
The venom of the North American pit viper is directly abnormal several days later. Late hematologic toxicity
destructive to local tissues, causes increased capillary is characterized by the development of new abnormal
permeability, and inhibits platelets and fibrinogen. hematologic laboratory parameters at least 4 days
Some species contain a neurotoxic component. The after the envenomation.45
exact venom composition varies among different spe-
cies of snakes and even differs within a given snake Clinical Presentation
species, depending on the time of year, the age of the Nausea, vomiting, and tachycardia may occur shortly
snake, and its geographic location. after a Crotalinae bite. Hypotension may occur and
may be multifactorial in origin (pre-existing dehydra-
Crotalinae tion and third-spacing of fluids from increased capillary
Crotalinae snakes have long, highly mobile, hollow permeability). While rare, hypotension can also occur
fangs that permit the snake to strike its prey and quickly in the setting of anaphylactoid reactions, and other
retreat. In approximately 75% of cases, venom is de- manifestations of anaphylactoid reactions, such as
posited in the subcutaneous tissue; the remaining 25% bronchospasm and urticarial rash, may be present.
of bites are “dry bites” in which no venom is injected. Typically, within a few hours of venom deposition,
Because venom can be deposited through either fang, pain and edema occur. However, the onset of edema
clinically relevant envenomations can occur even if there may be delayed or less impressive in children than
is only a single puncture wound. Conversely, because in adults, especially in children with lower extrem-
the snakes also have small teeth, some individuals will ity envenomations. Following venom deposition,
have more than 2 puncture wounds noted. the venom is absorbed and transported through
Despite receiving a disproportionate amount of the lymphatic system. Individuals will often develop
venom compared to body weight, pediatric patients tenderness over the proximal lymph nodes (eg, the
do not present significantly differently from adults, femoral lymph nodes in lower extremity bites and
nor do they experience a difference in treatment the axillary lymph nodes in upper extremity bites).
parameters or complications.41 Patients envenomated by pit vipers often complain
of a metallic taste in their mouth.42 Localized neuro-
Pathophysiology muscular effects (eg, fasciculations and myokymia)
The Mojave rattlesnake can contain either of 2 distinct may be present, depending on the specific species of
types of venom: A or B. Snakes that contain venom snake. Local oozing is often noted from the puncture
A (which contains Mojave toxin) have a curare-like wounds. However, even with associated profoundly
mechanism of action that can result in a nondepolariz- abnormal laboratory values, clinically significant hem-
ing neuromuscular blockade. Clinically, a descending orrhage is very uncommon.

GROUP SUBSCRIPTIONS: groups@ebmedicine.net 11 © 2021 EB MEDICINE. ALL RIGHTS RESERVED.


The initial evaluation of a patient with a suspected documented elevated compartment pressures de-
pit viper envenomation should focus on a thorough his- spite administration of additional antivenom.6,42 Co-
tory and physical examination. Table 3 provides ques- agulopathy should be corrected prior to fasciotomy.
tions to ask while taking the history. Serial examinations Patients with coagulopathy, thrombocytopenia,
should be performed every 15 to 30 minutes to assess or significant progressive edema should receive
for progression of edema.42 Some experts advocate antivenom.6,42 There are now 2 antivenom products
for marking the leading edge by drawing a line on the approved for use in the United States: Crotalidae
skin and assessing when and where the edema stops. polyvalent immune Fab (CroFab®), FDA-approved for
Because this method is often subjective, others advo- use in 2000; and Crotalidae equine immune F(ab’)2
cate for serial measurements of the circumference of (Anavip®), FDA-approved for use in 2015. Both
the mid-foot, mid-calf, and mid-thigh (in lower extrem- antivenoms are approved for the treatment of rattle-
ity bites), or mid-hand, mid-forearm, and mid-upper snake, copperhead, and cottonmouth bites.
arm (in upper extremity bites). When using the latter CroFab® is a lypholized powder that requires slow
method, it is important to mark the area measured, so reconstitution with gentle stirring to avoid formation
serial measurements are taken at the same location. of bubbles and to avoid the breakdown/denaturing of
Laboratory studies (including a CBC, prothrombin the protein, which would render it useless. CroFab®
time, and fibrinogen level) should be obtained. The is made from the venom of 4 snake species: Crotalus
CBC may reveal thrombocytopenia, but it may also re- atrox (Western diamondback), C adamanteus (Eastern
veal elevated hemoglobin due to hemoconcentration. diamondback), C scutulatus (Mojave rattlesnake), and
In the absence of any abnormalities on the initial blood Agkistrodon piscivorus (cottonmouth). Four flocks
work or physical examination, these laboratory param- of sheep are immunized utilizing the venom of the
eters should be repeated in 6 to 8 hours. Patients may different snakes. The serum is then obtained, and
also develop early hematologic toxicities that may be the antibodies are separated from the serum. Papain
present on admission or within the first several hours is used to cleave the antibody to obtain the Fab
after envenomation. Such hematologic abnormalities fragment. Adult and pediatric patients who present
include either thrombocytopenia and/or coagulopathy, with hemodynamic instability or other immediate
which can be characterized by a low fibrinogen and or life-threatening events should receive 8 to 12 vials of
an elevated prothrombin time. CroFab® antivenom.43 For all other patients with evi-
dence of toxicity, an initial dose of 4 to 6 vials of Cro-
Treatment Fab® antivenom should be administered, and repeat
The envenomated limb should be splinted in extension laboratory tests should be obtained 1 hour after the
and elevated at the level of the heart. Pressure immo- infusion is complete. If laboratory parameters worsen
bilization bandages are not recommended for man- or if edema progresses, the initial dose should be
agement of North American Crotalinae snake bites.36 repeated. The current FDA-approved dosing regimen
Tourniquets, ice, and suction devices were once con- recommends 2 additional vials of antivenom admin-
sidered desirable field treatments for North American istered at 6, 12, and 18 hours after initial control is
snake bites, but mounting evidence discourages their achieved. Many toxicologists have moved away from
use in the field or in the ED. Analgesics should be routine maintenance dosing, and rather employ an
administered liberally, and opioids are preferred. Due “as needed” approach, in which scheduled mainte-
to lack of histamine release, fentanyl (1-2 mcg/kg IV) nance doses are not administered, and additional
is often considered the preferred opioid. Nonsteroidal antivenom is administered based on progression of
anti-inflammatory drugs (NSAIDs) should be avoided edema or worsening laboratory parameters.46 There is
due to their antiplatelet effects. no dosing adjustment required for pediatric patients.
While extremity edema can be impressive and Anavip® is made from the venom of the snakes
may be difficult to distinguish from compartment Bothrops asper (fer-de-lance) and C durissus (South
syndrome, true compartment syndrome following American rattlesnake). Horses are immunized utilizing
North American pit viper bites is very uncommon. the venom, and antibodies are then isolated from the
Fasciotomy should not be performed without clearly serum. Pepsin is used to cleave the antibody to obtain
a V-shaped F(ab’)2 fragment. Anavip® reconstitutes
more quickly than CroFab®, has been shown to reduce
Table 3. Obtaining the History of Snake late hematologic toxicities, and requires no routine
Bites maintenance dosing compared to CroFab®, although
• Did you see the snake? overall comparative data are quite limited.3,47 The
• Did the snake have any distinct markings? current FDA-approved regimen for Anavip® dosing is
• Did the snake have a rattle? 10 vials administered initially for all patients with toxic-
• What were you doing when you were bitten?
ity, with monitoring and retesting parameters similar
• Have you previously been envenomated by a snake?
• If so, did you receive antivenom?
to CroFab®. Dosing of Anavip® should be repeated
www.ebmedicine.net (with 4 vials) as necessary to achieve control of enven-

SEPTEMBER 2021 • www.ebmedicine.net 12 ©2021 EB MEDICINE


omation; once symptoms and laboratory findings stops the progression of edema but does not reverse
stabilize, no additional dosing is routinely neces- edema, the edema that is present may last up to sev-
sary.48 All patients who receive either antivenom eral weeks. Furthermore, upon discharge, if patients
should also receive an IV crystalloid fluid bolus of at put the extremity in a dependent position, edema
least 20 mL/kg. Patients with copperhead bites often and pain will likely increase. Upon discharge, patients
require only a single dose of 4 vials of CroFab®,42 should be given an adequate amount of analgesics,
but patients can be redosed in the rare instance of but NSAIDs should be avoided. Patients with evidence
recurrent symptoms. of toxicity, based on either the physical examination
The initial dose of antivenom should be admin- or laboratory parameters, should be admitted to a
istered in a minimum volume of 250 mL of normal monitored setting for at least 24 hours. Patients with
saline. The infusion should begin at 25 mL/hr and be moderate and progressive or severe toxicity should
doubled steadily every 5 to 10 minutes, such that the be admitted to an intensive care setting for frequent
full infusion should be completed in just over 1 hour. laboratory checks and close monitoring.
With each rate increase, evidence of anaphylactoid Because of the risk for late hematologic toxicity,
reaction should be assessed. If an anaphylactoid patients who receive antivenom, or who have mani-
reaction occurs, the infusion should be stopped, and festations of toxicity, should have laboratory param-
1 mg/kg of IV diphenhydramine should be adminis- eters obtained at 3 to 5 days and at 5 to 7 days after
tered. If continuation of the antivenom is indicated, the last dose of antivenom.6 Retreatment thresholds
the infusion should be restarted at a lower rate, with are complex, and should be discussed with a medical
concurrent infusion of epinephrine, if required. While toxicologist, especially because patients with late he-
the rates of anaphylactoid reactions are much lower matologic toxicity often respond poorly to additional
with CroFab® antivenom than with the previously antivenom. Patients with manifestation of an enven-
used Wyeth antivenom, it is still recommended that omation should be admitted to the hospital.
antihistamines, corticosteroids, and epinephrine
should be immediately available when administering Elapidae
antivenom. Overall, adverse events are similar be- Coral snakes are the only venomous elapids that
tween CroFab® and Anavip®.3,47 are indigenous to North America. These snakes
Unless a patient with a pit viper bite is actively are brightly colored and easily identifiable by their
bleeding or requires an emergent surgical procedure, alternating red, yellow, and black color bands. Coral
platelets should not be transfused, as the toxin will snakes are often confused with the nontoxic king
destroy them as well as the patient’s native platelets. snake. However, several features can be used to
Early thrombocytopenia should resolve with antive- distinguish these snakes. The red band touches the
nom administration. When antivenom is administered yellow band in the coral snake, while the red band
for swelling, the limb should not be expected to touches the black band in the king snake, leading to
return to normal size at the time of treatment; the the well-known adage, “Red on yellow kills a fellow.
antivenom should halt progression of swelling only. Red on black, venom lack.” (See Figures 7 and 8,
In addition to antivenom administration, ad- page 14.) Furthermore, coral snakes have a black
equate supportive care should be ensured. Patients snout, while king snakes have a red snout.
should have local wound care, including debridement Among the coral snakes, the Sonoran coral snake
of any hemorrhagic blebs that form. Tetanus vaccina- (Micruroides euryxanthus), found in the Sonoran Des-
tions should be updated, as indicated. Due to the in- ert of Arizona and northern Mexico and in the south-
nate antimicrobial properties of venom, prophylactic west corner of New Mexico, is generally not consid-
antibiotics are not recommended. ered a medically important snake, as no case of severe
toxicity has been described following envenomation
Disposition by this snake. In contrast, the Eastern coral snake (M
Patients who have an apparent dry bite (no cutane- fulvius) and the Texas coral snake (M tener) are much
ous signs of edema and normal laboratory studies at more potent.
both initial presentation and 6-8 hours later) can be
discharged home. Other patients without any evidence Pathophysiology
of toxicity based on examination or laboratory param- Unlike the fangs of rattlesnakes, coral snakes have
eters should be observed for 8 hours and then con- small, fixed, rear teeth, requiring the snake to remain
sidered for discharge to home. Pediatric patients with attached for up to several seconds in order for venom
lower extremity bites are at an increased risk for de- delivery to occur. Consequently, the snake is unable
layed edema, so they should be observed for 12 to 24 to strike rapidly; rather, it must “chew” on the victim
hours.41,42 Observation for 24 hours should be strongly in order to inject venom.
considered for pediatric patients with lower extremity Coral snake bites typically do not result in significant
bites due to the difficulty in appreciating subtle early tissue destruction. However, the neurotoxic effects
edema. It should be noted that, because antivenom that do develop may do so in a delayed fashion.49

GROUP SUBSCRIPTIONS: groups@ebmedicine.net 13 © 2021 EB MEDICINE. ALL RIGHTS RESERVED.


Neurologic effects include ptosis, bulbar muscle Treatment
weakness, fasciculations, and, ultimately, respiratory There are no data demonstrating benefit from the use
arrest; the proximate cause of death in most coral of pressure immobilization bandages in clinical prac-
snake envenomations is respiratory arrest. Respiratory tice with North American neurotoxic elapid envenom-
paralysis can be delayed for up to 12 hours.49 Therefore, ations; however, in unique situations in which there
all patients bitten by coral snakes, except the Sonoran is a significant delay to medical attention, pressure
coral snake, should be admitted for observation. immobilization bandages may be considered. Caution
Thrombocytopenia or coagulopathy should not be should be used, since systemic absorption may be
expected following coral snake envenomation. increased if the bandage is applied too tightly.
In the past, it was thought that all patients bitten
by a coral snake should be admitted and receive
antivenom. However, Wyeth Pharmaceuticals discon-
tinued production of North American coral snake anti-
Figure 7. Coral Snake (Micrurus fulvius) venom, and supplies are limited. For the past several
years, the FDA has extended the expiration dates for
the available antivenom due to the limited supplies.
Thus, one approach is to observe all patients with
potential coral snake bites in a monitored setting for
24 hours. Antivenom should be reserved for patients
with neurologic abnormalities. At the first indication
of muscle weakness, 3 to 5 vials of antivenom should
be administered in 250 mL of normal saline (0.9%
sodium chloride). If respiratory insufficiency develops
and no antivenom is available, endotracheal intuba-
tion may be required.
A phase III study (NCT01337245) evaluating a
new antivenom for the treatment of coral snake bites
(an F(ab’)2 fragment; anticipated brand name Cor-
almyn®) has been conducted recently, though the
results have not yet been released.

Disposition
All patients bitten by a coral snake should be admit-
Source: https://commons.wikimedia.org/wiki/File:Coral_009.jpg. Author:
Norman.benton. Used under the Creative Commons Attribution-Share
ted to the hospital to monitor for possible develop-
Alike 3.0 Unported license. https://creativecommons.org/licenses/by- ment of neurologic toxicity.
sa/3.0/deed.en

Figure 8. Scarlet King Snake (Lampropeltis


n Venomous Lizards
elapsoides) Epidemiology
There are 2 venomous North American lizards: the
Gila monster (Heloderma suspectum), found in the
desert in the southwestern United States (eg, Arizona,
New Mexico), and the beaded lizard (H horridum),
primarily found in Mexico. Because these animals are
nocturnal and shy, and primarily bite only when being
handled, human bites from these lizards are relatively
uncommon.

Pathophysiology
The venom contains a mixture of enzymes including
phospholipase A2, hyaluronidase, serotonin, gilatoxin,
and vasoactive peptides. The hyaluronidase likely per-
mits the remainder of the venom to spread throughout
the tissues. The gilatoxin is responsible for inducing
Source: https://commons.wikimedia.org/wiki/File:Scarlet_King_ hypotension and increasing bradykinin levels.
Snake_(Lampropeltis_elapsoides)_(32391807822).jpg. Author: Peter
Paplanus. Used under the Creative Commons https://en.wikipedia.
org/wiki/en:Creative_Commons Attribution 2.0 Generic license https://
creativecommons.org/licenses/by/2.0/deed.en.

SEPTEMBER 2021 • www.ebmedicine.net 14 ©2021 EB MEDICINE


Clinical Presentation gists (experts at managing snake bite patients) found
The lizards are known to latch on to their victim and shorter lengths of stay and less antivenom used in
are often difficult to remove. The patient may com- the group that was given additional antivenom on an
plain of severe pain. Hypotension and angioedema as-needed basis compared to the group that received
are often encountered. Severe leukocytosis is com- routine maintenance dosing.46 However, the external
mon. No specific diagnostic tests are recommended, validity of that study has been questioned.
but radiographs may be indicated to assess for
retained teeth. Ideal Antivenom Dosing for Patients With
Copperhead or Cottonmouth Envenomation
Treatment There is also some question regarding the ideal dosing
Treatment is largely supportive, as there is no an- of antivenom for patients with copperhead or cotton-
tivenom available. Hypotension should be treated mouth envenomations. Use of the same dosing recom-
first with an IV crystalloid bolus, and if hypotension mendations as for treatment of rattlesnakes has been
persists, direct-acting vasopressors (eg, epinephrine questioned. While hematologic toxicity can occur, it is
or norepinephrine) may be indicated. Corticosteroids typically characterized only by pain and edema.
and diphenhydramine may be indicated for treatment
of angioedema. Patients with angioedema should be Anavip® Antivenom
intubated if there is concern for airway compromise. In April 2021, Anavip® was approved for the treat-
Since these lizards “chew” versus strike, the ment of all North American pit viper envenomations.
wound should be examined for embedded teeth. Previously, it had FDA approval only for treatment of
Prophylactic antibiotics may be needed. North American rattlesnakes. Now, with the expand-
ed approval, it can be used for treatment of copper-
Disposition heads and cottonmouths, in addition to rattlesnakes.
Patients should be observed for 6 hours after the bite.
If there are no signs of toxicity, the patient can be dis-
charged. Angioedema warrants admission to the ICU. n Time- and Cost-Effective Strategies
• Antivenom should not be administered empiri-
cally for all Crotalinae envenomations. Antivenom
n Special Populations should be administered only when there are
Pediatric patients, elderly patients, and debilitated laboratory abnormalities (eg, thrombocytopenia,
patients may be at higher risk for developing severe coagulopathy, hypofibrinogenemia) or in the set-
systemic manifestations after a fire ant attack. Pedi- ting of progressive swelling.
atric patients are considered more likely to develop • Given the limited supply of antivenom, coral
high-grade envenomations following scorpion stings, snake antivenom should not be administered
likely because they receive a higher amount of venom empirically unless there is a manifestation of
on a per-kilogram basis. Patients on anticoagulants or neurologic compromise. Once the first sign or
antiplatelet agents are at greater risk for hemorrhagic symptom of toxicity develops, antivenom should
complications following rattlesnake envenomation.50 be administered.
In addition, pregnant women may be at increased risk • Fasciotomy should not be performed for pit viper
for toxicity after a snake bite.51 bites without clearly documented elevated com-
partment pressures that persist despite elevation
of the extremity and administration of additional
n Controversies and Cutting Edge antivenom.
Anascorp® Dosing
The exact dosing of Anascorp® is controversial. The
FDA-approved dosing is 3 vials for patients with high- n Summary
grade envenomations (grade III or IV). However, recent While bites and stings from venomous animals are
data indicate that 1 vial may suffice, without additional somewhat rare, it is imperative that emergency clini-
complications or adverse clinical outcomes.37 cians correctly diagnose the bite or sting to allow for
proper treatment to be administered. Emergency
Snake Bite Antivenom Dosing clinicians should be able to identify the envenomations
CroFab® Maintenance Dosing that require antivenom and those that can be treated
There is much debate about the need for routine main- with supportive care or other methods. Knowledge
tenance dosing of CroFab®. While some literature has of current information on the treatment of pediatric
demonstrated that routine maintenance dosing may envenomations is key to proper care and disposition
not be needed, others advocate for its routine use. A of these patients. Patients with black widow spider
study in Phoenix in which rattlesnake-envenomated pa- bites may present with severe abdominal or back pain,
tients were treated at the bedside by medical toxicolo- along with fasciculations and diaphoresis. Treatment

GROUP SUBSCRIPTIONS: groups@ebmedicine.net 15 © 2021 EB MEDICINE. ALL RIGHTS RESERVED.


Risk Management Pitfalls for Terrestrial
Envenomations in Pediatric Patients

1. “I wanted to make the patient with the black 6. “The patient looked like he had a classic ana-
widow bite feel better without using opioids, phylactic reaction, but he had not been stung
so I gave him Latrodectus antivenom. He had before.” Anaphylactic reactions to venom have
an anaphylactoid reaction to the antivenom preformed antibodies, while massive envenom-
and felt worse!” Reserve black widow (L mac- ations can result in anaphylactoid reactions without
tans) antivenom for individuals with end-organ any prior exposure. However, the treatment is the
dysfunction or for those who have failed conser- same, and it should focus on ensuring airway pa-
vative management with opioids and benzodi- tency and using IM epinephrine, IV fluid boluses,
azepines. Antivenom should not be a first-line H1 and H2 antagonists, and corticosteroids.
therapy.
7. “To reduce absorption of the venom, we
2. “The patient was bitten by a black widow. I applied a tight tourniquet to the arm of the
administered calcium, but the symptoms did rattlesnake bite victim.” Do not apply ice, tour-
not improve.” Calcium is not recommended for niquets, suction devices, or other similar objects
patients with black widow bites, as data have not to the skin of a rattlesnake bite, as systemic ab-
demonstrated its efficacy. sorption may actually be increased. The enveno-
mated limb should be splinted in extension and
3. “This patient who resides in the Northeast elevated.
was diagnosed with a brown recluse bite 2
days ago. However, on examination today, 8. “The patient was bitten by a rattlesnake, but
there appears to be significant cellulitis around he had only minor pain, so we gave him ibu-
the bite.” Be extremely cautious about making profen.” Due to their antiplatelet effects, avoid
the diagnosis of a brown recluse bite in patients the use of NSAIDs for the treatment of pain as-
who live in areas outside the typical geographic sociated with pit viper envenomation.
distribution of the brown recluse spider. In such
situations, the lesions are much more likely to be 9. “My patient who was bitten by a rattlesnake
an abscess than a brown recluse bite. Do not mis- had impressive swelling and some skin necro-
take a necrotizing soft-tissue infection for a brown sis, so we administered prophylactic antibiot-
recluse bite. ics.” Do not administer prophylactic antibiotics
for a Crotalinae envenomation. Antibiotics should
4. “We removed 200 bee stingers from the child, be reserved for patients in whom clinical signs or
but he was still hypotensive.” Do not delay the symptoms of infection develop.
initial resuscitation of a patient who presents fol-
lowing a massive Hymenoptera envenomation to 10. “The patient said the snake that bit him had a
remove the stingers. Research has demonstrated combination of black, red, and yellow bands.
that 90% of a bee’s venom is injected within the The patient didn’t have any symptoms, so we
first 30 seconds after a sting, and virtually 100% is assumed it was a king snake.” Do not mistake
injected at 1 minute. Therefore, the initial man- a coral snake bite for a king snake bite simply
agement of victims of massive Hymenoptera en- because of a lack of symptoms during the initial
venomations should not focus on stinger removal. ED evaluation. If there is concern that a snake
may have been an elapid and the patient was
5. “The patient said he was stung by nearly 60 in an area where neurotoxic coral snakes reside,
bees, but he looked good after 4 hours. We prolonged observation may be required to ensure
assumed he was exaggerating, so we sent him that delayed neurotoxicity does not develop.
home. He returned in renal failure 2 days lat-
er.” Because of the potential for delayed onset of
many symptoms, a child who has ≥50 honeybee
stings (or ≥2 stings/kg for children weighing <25
kg) should be admitted to a monitored setting for
24 hours.

SEPTEMBER 2021 • www.ebmedicine.net 16 ©2021 EB MEDICINE


Case Conclusions
The 8-year-old boy who presented after a bite by a rattlesnake...
The patient’s laboratory test results were as follows: hemoglobin, 17.5 g/dL; platelets, 52x109/L; prothrombin
time, 58 seconds; and fibrinogen, <5 mg/dL. The patient was administered 6 vials of Crofab® antivenom;
CASE 1

however, despite the initial loading dose of antivenom, the edema progressed, although there was some
improvement in the laboratory parameters when they were checked again. Because of the worsening edema,
an additional 6 vials of antivenom were administered, and edema progression halted. Subsequent laboratory
values were improved. The patient was admitted overnight for serial examinations, follow-up laboratory tests,
and additional antivenom therapy. After discharge, he was seen twice in follow-up and did not develop any
evidence of recurrent thrombocytopenia or coagulopathy. The patient made a full recovery.

The 9-year-old girl who presented after she complained of a “pinch” on her arm while raking leaves...
The patient’s laboratory findings were unrevealing, except for mild leukocytosis. On examination, you noted
CASE 2

a bull’s eye rash, consistent with a black widow envenomation. The patient's condition improved with ben-
zodiazepines and opioids. You confirmed that the use of antivenom in this patient may be more dangerous
than the actual envenomation, so given the patient's improvements with treatment, you did not administer
the antivenom.

The 17-month-old boy who presented after awakening crying in his crib shortly after midnight...
The patient was noted to be tachycardic and hypertensive. He was agitated, screaming, and appeared to
CASE 3

have noncontrollable movements of his arms and legs. In addition, roving eye movements were noted.
There were no lesions on the skin, but a positive tap test was noted on the foot, confirming your suspicion
of a bark scorpion envenomation. The patient was given 3 vials of scorpion antivenom with resolution of his
symptoms, and was discharged home a few hours after presentation.

should consist of benzodiazepines and opioids, with ence, pertinent information about the study, such as the
antivenom being reserved for those who fail traditional type of study and the number of patients in the study is
measures. Brown recluse envenomation may result in included in bold type following the references, where
significant local tissue injury, but debridement should available. The most informative references cited in this
be delayed pending clear delineation of the extent paper, as determined by the authors, are noted by an
of necrosis. Corticosteroids can be administered for asterisk (*) next to the number of the reference.
systemic loxoscelism. Patients with Hymenoptera
envenomation may present with an anaphylactic reac- 1. Gummin DD, Mowry JB, Spyker DA, et al. 2018 Annual report
of the American Association of Poison Control Centers’ National
tion if there is prior exposure; massive envenomation
Poison Data System (NPDS): 36th annual report. Clin Toxicol
presents similarly and is anaphylactoid owing to the (Phila). 2019;57(12):1220-1413. (Retrospective database review)
large amount of melittin injected. Treatment is largely 2. Seifert SA, Warrick BJ. Immunotherapy. In: Brent J, Burkhart
supportive, but corticosteroids should be administered K, Dargan P, et al, ed. Critical Care Toxicology: Diagnosis and
for massive envenomations. A bark scorpion sting can Management of the Critically Poisoned Patient. 2nd ed. Cham:
cause significant neurotoxicity, especially in pediatric Springer; 1997:2843-2857. (Textbook chapter)
patients, and patients with high-grade envenomation 3.* Bush SP, Ruha AM, Seifert SA, et al. Comparison of F(ab’)2
may benefit from antivenom. Patients with snake bites versus Fab antivenom for pit viper envenomation: a prospec-
tive, blinded, multicenter, randomized clinical trial. Clin Toxicol
should be assessed carefully to determine whether (Phila). 2015;53(1):37-45. (Randomized controlled trial; 121
there are signs of toxicity or not (eg, a dry bite) and patients) DOI: 10.3109/15563650.2014.974263
administer antivenom appropriately. 4. Spyres MB, Lapoint J. Identification and management of marine
envenomations in pediatric patients. Pediatr Emerg Med Pract.
2020;17(4):1-24. (Review)
n References 5. Stoecker WV, Vetter RS, Dyer JA. NOT RECLUSE-a mnemonic
Evidence-based medicine requires a critical appraisal device to avoid false diagnoses of brown recluse spider bites.
JAMA Dermatol. 2017;153(5):377-378. (Review)
of the literature based upon study methodology and
6.* Levine M, Ruha AM, Graeme K, et al. Toxicology in the ICU:
number of subjects. Not all references are equally part 3: natural toxins. Chest. 2011;140(5):1357-1370. (Review)
robust. The findings of a large, prospective, random- DOI: 10.1378/chest.11-0295
ized, and blinded trial should carry more weight than 7. O’Connor AD, Ruha AM, Levine M. Pressure immobilization
a case report. bandages not indicated in the pre-hospital management of
To help the reader judge the strength of each refer- North American snakebites. J Med Toxicol. 2011;7(3):251.

GROUP SUBSCRIPTIONS: groups@ebmedicine.net 17 © 2021 EB MEDICINE. ALL RIGHTS RESERVED.


Clinical Pathway for Management of Rattlesnake
Envenomation

Patient presents following a


rattlesnake bite

• Obtain vital signs, CBC, PT,


fibrinogen levels
• Splint limb in extension and elevate

Signs of envenomation
(eg, edema, shock, or abnormal
laboratory findings)?

NO YES

• Perform serial examinations over the • Administer:


next 6 hours l
6 vials of CroFab®
• Update tetanus vaccination as OR
needed l
10 vials of Anavip®
• Administer analgesics (eg, 1-2 mcg/kg
fentanyl IV), as needed
• Avoid NSAIDs
• Repeat laboratory tests 1 hr after
completion of antivenom infusion

Did signs/symptoms of envenomation


develop during the 6 hours of
observation? Laboratory findings worsening or edema
progressing?

NO YES
YES NO

Recheck CBC, PT, fibrinogen


• Administer additional antivenom Admit for serial examinations, serial
based on original antivenom given: laboratory studies, and possible
l
6 vials of CroFab® additional antivenom
OR
l
4 vials of Anavip®
• Repeat laboratory tests
Evidence of thrombocytopenia and/or
coagulopathy?

NO YES

Observe in house if lower extremity


bite; otherwise, discharge home with
supportive care and return precautions.

Abbreviations: CBC, complete blood cell count; IV, intravenous; NSAIDs, nonsteroidal anti-inflammatory drugs; PT, prothrombin time.

SEPTEMBER 2021 • www.ebmedicine.net 18 ©2021 EB MEDICINE


(Position statement) 2004;116(12):843-846. (Case reports and review; 6 patients)
8. Hauke TJ, Herzig V. Dangerous arachnids-fake news or reality? 30. Kemp SF, deShazo RD, Moffitt JE, et al. Expanding habitat of
Toxicon. 2017;138:173-183. (Review) the imported fire ant (Solenopsis invicta): a public health con-
9.* Vetter RS. Clinical consequences of toxic envenomation by cern. J Allergy Clin Immunol. 2000;105(4):683-691. (Review)
spiders. Toxicon. 2018;152:65-70. (Review) 31.* Skolnik AB, Ewald MB. Pediatric scorpion envenomation in the
DOI: 10.1016/j.toxicon.2018.07.021 United States: morbidity, mortality, and therapeutic innova-
10. Levine M, Canning J, Chase R, et al. Cardiomyopathy following tions. Pediatr Emerg Care. 2013;29(1):98-103. (Review)
latrodectus envenomation. West J Emerg Med. 2010;11(5):521- DOI: 10.1097/PEC.0b013e31827b5733
523. (Case report; 1 patient) 32. Curry SC, Vance MV, Ryan PJ, et al. Envenomation by the
11. Murphy CM, Hong JJ, Beuhler MC. Anaphylaxis with Latro- scorpion Centruroides sculpturatus. J Toxicol Clin Toxicol.
dectus antivenin resulting in cardiac arrest. J Med Toxicol. 1983;21(4-5):417-449. (Review)
2011;7(4):317-321. (Review) 33. Stahnke HL. Arizona’s lethal scorpion. Ariz Med. 1972;29(6):490-
12. Isbister GK, Fan HW. Spider bite. Lancet. 2011;378(9808):2039- 493. (Review)
2047. (Review) 34. Coorg V, Levitan RD, Gerkin RD, et al. Clinical presentation
13. Quan D. North American poisonous bites and stings. Crit Care and outcomes associated with different treatment modali-
Clin. 2012;28(4):633-659. (Review) ties for pediatric bark scorpion envenomation. J Med Toxicol.
2017;13(1):66-70. (Retrospective study; 156 patients)
14. Offerman SR, Daubert GP, Clark RF. The treatment of black wid-
ow spider envenomation with antivenin Latrodectus mactans: a 35. Patel A, Elston DM. What’s eating you? Bark scorpions (Cen-
case series. Perm J. 2011;15(3):76-81. (Case series; 4 patients) truroides exilicauda and Centruroides sculpturatus). Cutis.
2020;105(5):239-240. (Review)
15.* Dart RC, Bush SP, Heard K, et al. The efficacy of antivenin
Latrodectus (black widow) equine immune F(ab’)(2) versus 36. O’Connor A, Ruha AM. Clinical course of bark scorpion
placebo in the treatment of latrodectism: a randomized, envenomation managed without antivenom. J Med Toxicol.
double-blind, placebo-controlled, clinical trial. Ann Emerg 2012;8(3):258-262. (Retrospective chart review; 88 patients)
Med. 2019;74(3):439-449. (Randomized controlled trial; 60 37. Quan D, LoVecchio F, Bhattarai B, et al. Comparing clinical
patients) DOI: 10.1016/j.annemergmed.2019.02.007 outcomes between two scorpion antivenom dosing strategies
16. Clark RF, Wethern-Kestner S, Vance MV, et al. Clinical presen- in children. Clin Toxicol (Phila). 2019;57(9):760-764. (Retrospec-
tation and treatment of black widow spider envenomation: tive review; 141 children)
a review of 163 cases. Ann Emerg Med. 1992;21(7):782-787. 38. World Health Organization. Venomous snakes distribution and
(Retrospective study; 163 patients) species risk categories. Available at: http://apps.who.int/blood-
17. Lopes PH, Squaiella-Baptistão CC, Marques MOT, et al. products/snakeantivenoms/database. Accessed August 15,
Clinical aspects, diagnosis and management of Loxosceles 2021. (Government report)
spider envenomation: literature and case review. Arch Toxicol. 39. National Institute for Occupational Safety and Health. Venom-
2020;94(5):1461-1477. (Systematic review) ous snakes. Available at: https://www.cdc.gov/niosh/topics/
18. Furbee RB, Kao LW, Ibrahim D. Brown recluse spider envenom- snakes/default.html#:~:text=It%20has%20been%20estimat-
ation. Clin Lab Med. 2006;26(1):211-226. (Review) ed%20that,did%20not%20seek%20medical%20care. Accessed
August 15, 2021. (Government report)
19. Rosen JL, Dumitru JK, Langley EW, et al. Emergency depart-
ment death from systemic loxoscelism. Ann Emerg Med. 40. Ruha AM, Kleinschmidt KC, Greene S, et al. The epidemiology,
2012;60(4):439-441. (Case report; 1 patient) clinical course, and management of snakebites in the North
American Snakebite Registry. J Med Toxicol. 2017;13(4):309-
20. Hogan CJ, Barbaro KC, Winkel K. Loxoscelism: old obstacles,
320. (Review)
new directions. Ann Emerg Med. 2004;44(6):608-624. (Review)
41. Levine M, Ruha AM, Wolk B, et al. When it comes to snake-
21. Hughes RL. A fatal case of acute renal failure from evenoming
bites, kids are little adults: a comparison of adults and children
syndrome after massive bee attack: a case report and literature
with rattlesnake bites. J Med Toxicol. 2020;16(4):444-451.
review. Am J Forensic Med Pathol. 2019;40(1):52-57. (Case
(Retrospective chart review; 420 patients)
report and review)
42. Lavonas EJ, Ruha AM, Banner W, et al. Unified treatment algo-
22. Silva GBDJ, Vasconcelos AGJ, Rocha AMT, et al. Acute kidney
rithm for the management of crotaline snakebite in the United
injury complicating bee stings - a review. Rev Inst Med Trop Sao
States: results of an evidence-informed consensus workshop.
Paulo. 2017;59:e25. (Review)
BMC Emerg Med. 2011;11:2. (Database review and consen-
23.* Lovecchio F, Cannon RD, Algier J, et al. Bee swarmings in sus statement)
children. Am J Emerg Med. 2007;25(8):931-933. (Prospective
43. Corbett B, Clark RF. North American snake envenomation.
study; 19 patients) DOI: 10.1016/j.ajem.2007.02.006
Emerg Med Clin North Am. 2017;35(2):339-354. (Review)
24. Schmidt JO. Clinical consequences of toxic envenomations by
44.* Ruha AM, Curry SC, Albrecht C, et al. Late hematologic toxicity
Hymenoptera. Toxicon. 2018;150:96-104. (Review)
following treatment of rattlesnake envenomation with crotali-
25. Lee JA, Singletary E, Charlton N. Methods of honey bee dae polyvalent immune Fab antivenom. Toxicon. 2011;57(1):53-
stinger removal: a systematic review of the literature. Cureus. 59. (Retrospective chart review; 66 patients)
2020;12(5):e8078. (Systematic review) DOI: 10.1016/j.toxicon.2010.09.014
26. Villada G, Hafeez F, Ollague J, et al. Imported fire ant enven- 45. Seifert SA, Mascarenas DN, Fullerton L, et al. Unpredicted
omation: a clinicopathologic study of a recognizable form of late-, new-onset thrombocytopenia and hypofibrinogenemia
arthropod assault reaction. J Cutan Pathol. 2017;44(12):1012- in Fab antivenom-treated rattlesnake envenomation. Toxicon.
1017. (Case series; 7 patients) 2020;184:55-56. (Case series; 3 patients)
27. Fitzgerald KT, Flood AA. Hymenoptera stings. Clin Tech Small 46. Spyres MB, Skolnik AB, Moore EC, et al. Comparison of antivenom
Anim Pract. 2006;21(4):194-204. (Review) dosing strategies for rattlesnake envenomation. Crit Care Med.
28. Goddard J, Jarratt J, de Castro FR. Evolution of the fire ant le- 2018;46(6):e540-e544. (Retrospective study; 310 patients)
sion. JAMA. 2000;284(17):2162-2163. (Case report; 1 patient) 47. Mascarenas DN, Fullerton L, Smolinske SC, et al. Comparison
29. deShazo RD, Kemp SF, deShazo MD, et al. Fire ant attacks on of F(ab’)(2) and Fab antivenoms in rattlesnake envenomation:
patients in nursing homes: an increasing problem. Am J Med. first year’s post-marketing experience with F(ab’)(2) in New

GROUP SUBSCRIPTIONS: groups@ebmedicine.net 19 © 2021 EB MEDICINE. ALL RIGHTS RESERVED.


Mexico. Toxicon. 2020;186:42-45. (Retrospective chart review;
37 patients)
4. Which of the following statements about the
48. Anavip Crotalidae immune F(ab')2 (equine). Available at:
® initial bite of a brown recluse spider is TRUE?
https://anavip-us.com/pi/. Accessed August 15, 2021. (Pack- a. It is very painful.
age insert) b. Necrosis can present within 6 hours.
49. Kitchens CS, Van Mierop LH. Envenomation by the Eastern c. Prompt administration of antivenom prevents
coral snake (Micrurus fulvius fulvius). A study of 39 victims. necrotic lesions from forming.
JAMA. 1987;258(12):1615-1618. (Retrospective chart review; d. The initial bite may be unnoticed due to a
39 patients)
lack of significant pain at the time of the bite.
50. Levine M, Ruha AM, Padilla-Jones A, et al. Bleeding following
rattlesnake envenomation in patients with preenvenomation use
of antiplatelet or anticoagulant medications. Acad Emerg Med. 5. What is the minimum number of bee stings to
2014;21(3):301-307. (Retrospective chart review; 319 patients) mandate admission?
51. Moore EC, Porter LM, Ruha AM. Rattlesnake venom-induced a. 5
recurrent coagulopathy in first trimester pregnant women - two b. 10
cases. Toxicon. 2019;163:8-11. (Case report and literature c. 25
review)
d. 50

n CME Questions 6. Which of the following can help identify a scor-


Current subscribers receive CME credit pion sting?
absolutely free by completing the follow- a. A large bull’s eye lesion
ing test. Each issue includes 4 AMA PRA b. Lymphatic streaks traveling away from the
Category 1 CreditsTM, 4 ACEP Category I sting site
credits, 4 AAP Prescribed credits, or 4 c. A tap test
AOA Category 2-A or 2-B credits. Online testing is d. Application of tap water
available for current and archived issues. To receive
your free CME credits for this issue, scan the QR 7. Following a scorpion sting, a 3-year-old boy
code below with your smartphone or visit has roving eye movements but no diffuse
www.ebmedicine.net/P0921 motor involvement. Which of the following
grades most accurately describes the grade of
envenomation of this patient?
a. Grade I
b. Grade II
c. Grade III
d. Grade IV

1. Which of the following prehospital therapies 8. The use of antivenom may be considered for
is recommended in the management of North treatment of which type of envenomation?
American pit viper bites? a. Massive honeybee envenomation
a. Application of ice b. Massive fire ant envenomation
b. Use of a commercial suction device c. Gila lizard bite
c. Splint and elevate d. Southern Pacific rattlesnake bite
d. Immersion in cold water
9. For a patient who received antivenom for a
2. Which of the following statements about black rattlesnake bite, which of the following repre-
widow envenomations is TRUE? sents the ideal follow-up strategy after dis-
a. The bite is typically painless. charge from the hospital?
b. Calcium chloride, but not gluconate, should a. Follow up at 1 to 3 days and at 10 days
be administered as soon as possible. b. Follow up at 3 to 5 days and at 5 to 7 days
c. Only the male spider is capable of causing c. Follow up at 2 weeks only
human envenomation. d. No follow-up appointment is required.
d. Pain, diaphoresis, or fasciculations can be
localized or diffuse. 10. Which of the following would be an indica-
tion for antivenom therapy after a coral snake
3. Which of the following is frequently misdiag- envenomation?
nosed as a brown recluse bite? a. Dysphagia
a. Soft-tissue infection b. Thrombocytopenia
b. Stevens-Johnson syndrome c. Hyperfibrinoginemia
c. Gilbert disease d. Hypofibrinoginemia
d. Cushing ulcers

SEPTEMBER 2021 • www.ebmedicine.net 20 ©2021 EB MEDICINE


CME Information
In Upcoming issues of Pediatric Date of Original Release: September 1, 2021. Date
of most recent review: August 15, 2021. Termination
Emergency Medicine Practice date: September 1, 2024.
Accreditation: EB Medicine is accredited by the
Accreditation Council for Continuing Medical Education (ACCME) to
provide continuing medical education for physicians. This activity has
JUNE 2021 | VOLUME 18 | ISSUE 6
been planned and implemented in accordance with the accreditation
PEDIATRIC requirements and policies of the ACCME.

Emergency Medicine Practice Evidence-Based Education • Practical Application


Credit Designation: EB Medicine designates this enduring material
for a maximum of 4 AMA PRA Category 1 CreditsTM. Physicians
should claim only the credit commensurate with the extent of their
CLINICAL CHALLENGES: participation in the activity.
• Which patients are at low risk for
clinically important traumatic brain Specialty CME: Included as part of the 4 credits, this CME activity is
injury?
• Which patients need a cranial CT scan
eligible for 0.5 Trauma CME credits and 1 Pharmacology CME credit,
and which can be observed safely in the
ED without imaging?
subject to your state and institutional requirements.
• What are the risk factors for post– ACEP Accreditation: Pediatric Emergency Medicine Practice is
concussion syndrome?
• What discharge guidance should be
approved by the American College of Emergency Physicians for 48
given to patients with concussion? hours of ACEP Category I credit per annual subscription.
Authors AAP Accreditation: This continuing medical education activity
Madeline Joseph, MD, FACEP, FAAP
Professor of Emergency Medicine and Pediatrics, Emergency Department
has been reviewed by the American Academy of Pediatrics and is
Associate Dean for Inclusion and Equity, Emergency
Medicine Department, University of Florida College Assessment and Management acceptable for a maximum of 48 AAP credits. These credits can be
of Medicine-Jacksonville, Jacksonville, FL

Audrey Paul, MD, PhD of Pediatric Acute Mild applied toward the AAP CME/CPD Award available to Fellows and
Assistant Professor; Pediatric Emergency Medicine;
NYU Long Island School of Medicine, New York, NY Traumatic Brain Injury and Candidate Fellows of the American Academy of Pediatrics.
Peer Reviewers Concussion AOA Accreditation: Pediatric Emergency Medicine Practice is
Susan B. Kirelik, MD, FAAP
Pediatric Emergency Physician, Rocky Mountain
n Abstract eligible for up to 48 American Osteopathic Association Category 2-A
Hospital for Children; Medical Director, Rocky
Mountain Pediatric OrthoONE Center for
Mild traumatic brain injury (mTBI) and concussion, a subtype of
mTBI, commonly present to the emergency department (ED)
or 2-B credit hours per year.
Needs Assessment: The need for this educational activity was
Concussion, Denver, Colorado and may present with symptoms identical to those associated
Todd W. Lyons, MD, MPH with more severe TBI. The development and use of clinical de-
Assistant Professor of Pediatrics and Emergency
Medicine, Harvard Medical School, Division of
cision rules, increased awareness of the risk of radiation associ-
ated with head computed tomography, and the potential for determined by a survey of medical staff, including the editorial board
Emergency Medicine, Boston Children’s Hospital,
Boston, MA
patient observation has allowed emergency clinicians to make
well-informed decisions regarding the need for imaging for pa-
of this publication; review of morbidity and mortality data from the
Prior to beginning this activity, see
tients who present with mTBI. For patients who present to the
ED with concussion, appropriate diagnosis, management, and
CDC, AHA, NCHS, an ACEP; and evaluation of prior activities for
“CME Information” on page 23. education are critical for optimal recovery. This issue reviews
the most recent literature on concussion and mTBI and provides
emergency physicians.
recommendations for the evaluation, diagnosis, and treatment
of mTBI and concussion in the acute setting. Target Audience: This enduring material is designed for emergency
medicine physicians, physician assistants, nurse practitioners, and
For online access, scan with your
residents.
smartphone camera or QR code reader app:

This issue is eligible for 4 CME credits. See page 23. EBMEDICINE.NET
Goals: Upon completion of this activity, you should be able to: (1)
demonstrate medical decision-making based on the strongest clinical
evidence; (2) cost-effectively diagnose and treat the most critical ED
presentations; and (3) describe the most common medicolegal pitfalls
• Transplant Patients for each topic covered.
CME Objectives: Upon completion of this activity, you should be
• Human Trafficking able to: (1) identify the common presentations of bites and stings
from various venomous terrestrial animals in North America; (2)
• Travel-Related Infections review indications for antivenom therapy to treat envenomations
in North America; and (3) determine the minimal amount of time a
patient should be observed in the emergency department before
Don't miss these practice-changing discharge.
topics coming soon! Discussion of Investigational Information: As part of the
journal, faculty may be presenting investigational information
about pharmaceutical products that is outside Food and Drug
Administration approved labeling. Information presented as part of
this activity is intended solely as continuing medical education and is
Group discounts for you not intended to promote off-label use of any pharmaceutical product.
and your colleagues Faculty Disclosure: It is the policy of EB Medicine to ensure objectivity,
balance, independence, transparency, and scientific rigor in all CME
activities. All faculty participating in the planning or implementation of
a CME activity are expected to disclose to the participants any relevant
EB Medicine offers budget-friendly discounts for financial relationships and to assist in resolving any conflict of interest
that may arise from the relationship. In compliance with all ACCME
group practices, hospitals and medical systems, accreditation requirements and policies, all faculty for this CME activity
academic medical centers, and other institutions were asked to complete a full financial disclosure statement. The
information received is as follows: Dr. Levine, Dr. Friedman, Dr. Feng,
with 5 or more clinicians. Our variety of formats, Dr. Gerber, Dr. Mishler, Dr. Skrainka, Dr. Claudius, Dr. Horeczko, and
CME tracking, and customized reporting makes their related parties report no relevant financial interest or other
EB Medicine the resource both administrators and relationship with the manufacturer(s) of any commercial product(s)
discussed in this educational presentation.
clinicians will value and trust. Commercial Support: This issue of Pediatric Emergency Medicine
Practice did not receive any commercial support.
For more information, contact Dana Stenzel, Earning CME Credit: Go online to www.ebmedicine.net/CME and
click on the title of the test you wish to take. When completed, a
Account Executive, at danas@ebmedicine.net CME certificate will be emailed to you.
or 678-336-8466, ext. 120, or visit Hardware/Software Requirements: You will need a Macintosh or PC
www.ebmedicine.net/groups with Internet capabilities to access the website.
Additional Policies: For additional policies, including our statement
of conflict of interest, source of funding, statement of informed
To learn more or subscribe, visit consent, and statement of human and animal rights, visit http://www.
www.ebmedicine.net/PEMPinfo ebmedicine.net/policies.

GROUP SUBSCRIPTIONS: groups@ebmedicine.net 21 © 2021 EB MEDICINE. ALL RIGHTS RESERVED.


Coming
this Fall to Evidence-Based

your smart Education - Practical


Application

device

Access Your Content


on the New
EB Medicine App

Podcasts Points & Full


Pearls Courses

SEPTEMBER 2021 • www.ebmedicine.net 22 ©2021 EB MEDICINE


The Pediatric Emergency Medicine Practice Editorial Board
EDITORS-IN-CHIEF Marianne Gausche-Hill, MD, FACEP, Melissa Langhan, MD, MHS Adam E. Vella, MD, FAAP
FAAP, FAEMS Associate Professor of Pediatrics and Associate Professor of Emergency Medicine
Ilene Claudius, MD Emergency Medicine; Fellowship Director, and Pediatrics, Associate Chief Quality
Medical Director, Los Angeles County EMS
Associate Professor; Director, Process & Director of Education, Pediatric Emergency Officer, New York-Presbyterian/Weill Cornell
Agency; Professor of Clinical Emergency
Quality Improvement Program, Harbor- Medicine, Yale University School of Medicine, New York, NY
Medicine and Pediatrics, David Geffen
UCLA Medical Center, Torrance, CA Medicine, New Haven, CT
School of Medicine at UCLA; Clinical
David M. Walker, MD, FACEP, FAAP
Tim Horeczko, MD, MSCR, FACEP, Faculty, Harbor-UCLA Medical Center,
Robert Luten, MD Chief, Pediatric Emergency Medicine,
Department of Emergency Medicine, Los
FAAP Professor, Pediatrics and Emergency Department of Pediatrics, Joseph M. Sanzari
Angeles, CA
Associate Professor of Clinical Emergency Medicine, University of Florida, Children’s Hospital, Hackensack University
Medicine, David Geffen School of Medicine, Michael J. Gerardi, MD, FAAP, Jacksonville, FL Medical Center; Associate Professor of
UCLA; Core Faculty and Senior Physician, Pediatrics, Hackensack Meridian School of
FACEP, President
Los Angeles County-Harbor-UCLA Medical Garth Meckler, MD, MSHS Medicine, Hackensak, NJ
Associate Professor of Emergency Medicine,
Center, Torrance, CA Associate Professor of Pediatrics, University
Icahn School of Medicine at Mount Sinai;
of British Columbia; Division Head, Pediatric Vincent J. Wang, MD, MHA
EDITORIAL BOARD Director, Pediatric Emergency Medicine,
Emergency Medicine, BC Children's Professor of Pediatrics and Emergency
Goryeb Children's Hospital, Morristown
Hospital, Vancouver, BC, Canada Medicine; Division Chief, Pediatric
Jeffrey R. Avner, MD, FAAP Medical Center, Morristown, NJ
Emergency Medicine, UT Southwestern
Chairman, Department of Pediatrics, Joshua Nagler, MD, MHPEd
Sandip Godambe, MD, PhD Medical Center; Director of Emergency
Professor of Clinical Pediatrics, Associate Division Chief and Fellowship
Chief Quality and Patient Safety Officer, Services, Children's Health, Dallas, TX
Maimonides Children's Hospital of Director, Division of Emergency Medicine,
Brooklyn, Brooklyn, NY Professor of Pediatrics, Attending Physician
Boston Children's Hospital; Associate INTERNATIONAL EDITOR
of Emergency Medicine, Children's Hospital
Professor of Pediatrics and Emergency
Steven Bin, MD of The King's Daughters Health System, Lara Zibners, MD, FAAP, FACEP,
Medicine, Harvard Medical School,
Associate Clinical Professor, UCSF School Norfolk, VA MMEd
Boston MA
of Medicine; Medical Director, Pediatric Honorary Consultant, Paediatric Emergency
Emergency Medicine, UCSF Benioff Ran D. Goldman, MD James Naprawa, MD Medicine, St. Mary's Hospital Imperial
Children's Hospital, San Francisco, CA Professor, University of British Columbia,
Attending Physician, Emergency College Trust, London, UK; Nonclinical
Pediatric Emergency Physician, BC
Department USCF Benioff Children's Instructor of Emergency Medicine, Icahn
Richard M. Cantor, MD, FAAP, Children’s Hospital, Vancouver, BC, Canada
Hospital, Oakland, CA School of Medicine at Mount Sinai, New
FACEP
Alson S. Inaba, MD, FAAP York, NY
Professor of Emergency Medicine and Joshua Rocker, MD
Pediatric Emergency Medicine Specialist,
Pediatrics; Section Chief, Pediatric Associate Chief and Medical Director, PHARMACOLOGY EDITOR
Kapiolani Medical Center for Women &
Emergency Medicine; Medical Director, Assistant Professor of Pediatrics and
Children; Associate Professor of Pediatrics, Aimee Mishler, PharmD, BCPS
Upstate Poison Control Center, Golisano Emergency Medicine, Cohen Children's
University of Hawaii John A. Burns School of Emergency Medicine Pharmacist, Program
Children's Hospital, Syracuse, NY Medical Center of New York, New Hyde
Medicine, Honolulu, HI Director – PGY2 Emergency Medicine
Park, NY
Steven Choi, MD, FAAP Pharmacy Residency, Valleywise Health
Madeline Matar Joseph, MD,
Chief Quality Officer and Associate Dean for Steven Rogers, MD Medical Center, Phoenix, AZ
Clinical Quality, Yale Medicine/Yale School FACEP, FAAP Associate Professor, University of
of Medicine; Vice President, Chief Quality Professor of Emergency Medicine and Connecticut School of Medicine, Attending
Officer, Yale New Haven Health System, Pediatrics, Associate Dean for Inclusion and Emergency Medicine Physician, Connecticut
New Haven, CT Equity, Emergency Medicine Department, Children's Medical Center, Hartford, CT
University of Florida College of Medicine-
Ari Cohen, MD, FAAP Jacksonville, Jacksonville, FL Jennifer E. Sanders, MD, FAAP,
Chief of Pediatric Emergency Medicine, FACEP
Massachusetts General Hospital; Instructor Anupam Kharbanda, MD, MSc Assistant Professor, Departments of
in Pediatrics, Harvard Medical School, Chief, Critical Care Services, Children's
Pediatrics, Emergency Medicine, and
Boston, MA Hospital Minnesota, Minneapolis, MN
Education, Icahn School of Medicine at
Tommy Y. Kim, MD Mount Sinai, New York, NY
Jay D. Fisher, MD, FAAP, FACEP
Clinical Professor of Emergency Medicine Health Sciences Clinical Professor of
Christopher Strother, MD
and Pediatrics, University of Nevada, Las Pediatric Emergency Medicine, University
Associate Professor, Emergency Medicine,
Vegas School of Medicine, Las Vegas, NV of California Riverside School of Medicine,
Pediatrics, and Medical Education; Director,
Riverside Community Hospital, Department
Pediatric Emergency Medicine; Director,
of Emergency Medicine, Riverside, CA
Simulation; Icahn School of Medicine at
Mount Sinai, New York, NY

CEO: Stephanie Williford


Director of Finance and Analytics: Robin Wilkinson
Publisher: Suzanne Verity
Director of Editorial Quality: Dorothy Whisenhunt, MS
Senior Content Editor & CME Director: Erica Scott
Direct all inquiries to: Content Editor: Cheryl Belton, PhD, ELS
Phone: 678-366-7933 Digital Editor & Project Manager: Angie Wallace
Fax: 770-500-1316 Editorial Assistant: Erin Mooney, MA
3475 Holcomb Bridge Road, Suite 100, Customer Service Representative: Kandis Slater
Account Executive: Dana Stenzel
Peachtree Corners, GA 30092
Director of Marketing: Anna Motuz, MBA
E-mail: ebm@ebmedicine.net Database Administrator: Jose Porras
Website: www.ebmedicine.net
ISSN info and disclaimer:
Pediatric Emergency Medicine Practice (ISSN Print: 1549-9650, ISSN Online: 1549-9669, ACID-FREE) is published monthly (12 times per year) by EB
Medicine (3475 Holcomb Bridge Road, Suite 100, Peachtree Corners, GA 30092). Opinions expressed are not necessarily those of this publication.
Mention of products or services does not constitute endorsement. This publication is intended as a general guide and is intended to supplement,
rather than substitute, professional judgment. It covers a highly technical and complex subject and should not be used for making specific medical
decisions. The materials contained herein are not intended to establish policy, procedure, or standard of care. Pediatric Emergency Medicine Practice
is a trademark of EB Medicine. Copyright © 2021 EB Medicine All rights reserved. No part of this publication may be reproduced in any format
without written consent of EB Medicine. This publication is intended for the use of the individual subscriber only, and may not be copied in whole or
in part or redistributed in any way without the publisher’s prior written permission – including reproduction for educational purposes or for internal
distribution within a hospital, library, group practice, or other entity.

GROUP SUBSCRIPTIONS: groups@ebmedicine.net 23 © 2021 EB MEDICINE. ALL RIGHTS RESERVED.


Points & Pearls
QUICK READ

Terrestrial Envenomations
in Pediatric Patients:
Identification and Management
SEPTEMBER 2021 | VOLUME 18 | ISSUE 9 in the Emergency Department

Points
• Envenomation by Latrodectus mactans (black Pearls
widow spiders) can result in significant pain and
l Massive Hymenoptera envenomations require
hyperadrenergic symptoms. Treatment should first admission and corticosteroid administration, even
be focused on ensuring adequate administration of if the patient is clinically improved after a short
opioids and benzodiazepines. period of observation.
• Use of Latrodectus antivenom is generally not con- l For children who have been bitten by a rattle-
sidered first-line therapy because of the relatively snake, edema in the lower extremities may be
high rate of immediate (anaphylactoid) or delayed delayed and difficult to appreciate. Consequently,
(serum sickness) allergic reactions. all children with lower extremity bites should be
• Because toxicity from Loxosceles (brown recluse) admitted and observed for 12 to 24 hours.
bites primarily manifests as necrotic dermal lesions,
various soft-tissue infections are often incorrectly l During administration of any antivenom (espe-
ascribed to envenomation from this spider. cially whole IgG molecules), careful monitoring for
• Patients envenomated by a Loxosceles spider anaphylactoid reactions is required.
should be observed for necrotic lesions and sys-
temic effects. Hemolysis should be treated with l Because of the antimicrobial aspects of snake
corticosteroids. venom, routine antibiotics are not indicated fol-
• Massive hymenoptera envenomation can result in lowing envenomation.
multisystem organ involvement. Treatment with
corticosteroids is indicated even though the reac-
tion is not IgE-mediated.
• Because of the potential for delayed onset of many • Patients who have a hematologic or cutaneous mani-
symptoms, a child who has ≥50 honeybee stings festation of toxicity following a rattlesnake bite can
(or ≥2 stings/kg for children weighing <25 kg) be treated with CroFab® or Anavip®.
should be admitted to a monitored setting for 24 • Due to their antiplatelet effects, NSAIDs should be
hours.10 For pediatric patients who have between 2 avoided for the treatment of pain associated with pit
and 50 stings (or <2 stings/kg), laboratory param- viper envenomation.
eters should be obtained at presentation and at 6 • Do not perform a fasciotomy for pit viper bites with-
hours after presentation. out clearly documented elevated compartment pres-
• Because bark scorpion venom lacks dermonecrotic sures that persist despite elevation of the extremity
components, the sting site is often not visible. In and administration of additional antivenom.
cases of diagnostic uncertainty, a tap test (in which • Given the current shortages, antivenom should be
tapping the envenomated area elicits a painful reserved for patients who have been bitten by a coral
reaction) can assist in establishing the diagnosis. snake who have symptoms, and it should be admin-
• Patients with symptoms other than pain following a istered at the first sign of symptoms, which is often
bark scorpion envenomation should be observed. mild ptosis.
Those with evidence of neurotoxicity should be • Gila monster envenomations are uncommon, due
assessed for the need for antivenom. In cases of to the shy nature of the animal. However, in cases
high-grade envenomation in which antivenom is not of envenomation, the animal often latches onto the
available, intensive care admission is often required. victim vigorously, and it may be difficult to remove.
• Patients with rattlesnake envenomations should The victim may develop angioedema.
have the extremity splinted in full extension and
elevated. Ice, tourniquets, and compression ban-
dages are not recommended.

SEPTEMBER 2021 • www.ebmedicine.net 24 ©2021 EB MEDICINE

You might also like